Discussion:
trying to understand/visualize EM waves. please help
(too old to reply)
akh2103
2010-05-30 23:46:46 UTC
Permalink
Hi--
I'm a recent humanities grad trying to get a grasp of EM. I can
understand/visualize sound waves propagating through a medium. But I
can't understand or visualize EM waves in the same way. I get that EM
waves are 'fundamental' and can't be broken down/explained in terms of
smaller parts (the way a sound wave can be analyzed in terms of
molecules). but i still can't 'see' EM waves. Can anybody help?

Thanks!
AH
akh2103
2010-05-31 00:01:43 UTC
Permalink
Here is a better way of explaining my question. EM waves are a
'disturbance.' What is being disturbed? Electromagnetic space itself?
Is there any way to 'see' this or draw an analogy with gravitational
disturbances?
Szczepan Bialek
2010-06-01 17:34:02 UTC
Permalink
Post by akh2103
Here is a better way of explaining my question. EM waves are a
'disturbance.' What is being disturbed? Electromagnetic space itself?
Is there any way to 'see' this or draw an analogy with gravitational
disturbances?
Electric waves are exactly like the sound waves. They both are the real
waves.
They are the linear vibrations of molecules (in gas, liquid or solids) or
electrons.
So the sound waves we can call "molecular waves" end the electric waves
"electron waves".

Maxwell's EM waves are the torsional (angle or rotational) vibrations. They
start as the oscillating magnetic whirl around the wire with the electric
current.

Such model was stripped away. But the math remain. Maxwell was a math
genius. So the math for EM is usefull in machinery with rotating parts.
http://en.wikipedia.org/wiki/Maxwell_material
S*
akh2103
2010-05-31 00:28:24 UTC
Permalink
Post by akh2103
Hi--
I'm a recent humanities grad trying to get a grasp of EM. I can
understand/visualize sound waves propagating through a medium. But I
can't understand or visualize EM waves in the same way. I get that EM
waves are 'fundamental' and can't be broken down/explained in terms of
smaller parts (the way a sound wave can be analyzed in terms of
molecules). but i still can't 'see' EM waves. Can anybody help?
Thanks!
AH
http://answers.yahoo.com/question/index?qid=20100504202743AACTiJT
blackhead
2010-05-31 01:59:23 UTC
Permalink
Post by akh2103
Hi--
I'm a recent humanities grad trying to get a grasp of EM. I can
understand/visualize sound waves propagating through a medium. But I
can't understand or visualize EM waves in the same way. I get that EM
waves are 'fundamental' and can't be broken down/explained in terms of
smaller parts (the way a sound wave can be analyzed in terms of
molecules). but i still can't 'see' EM waves. Can anybody help?
Thanks!
AH
The electromagnetic force in an EM wave is carried by photons that
possess energy and momentum that are related by Energy = (speed of
light) x momentum. Collectively, their vast number gives rise to EM
effects such as the propagation of EM waves, but their behaviour
individualy is governed by a branch of physics called Quantum
Electrodynamics which deals with their creation, propagation and
absorption in the vacuum.

Larry
Salmon Egg
2010-05-31 04:26:11 UTC
Permalink
In article
Post by akh2103
Hi--
I'm a recent humanities grad trying to get a grasp of EM. I can
understand/visualize sound waves propagating through a medium. But I
can't understand or visualize EM waves in the same way. I get that EM
waves are 'fundamental' and can't be broken down/explained in terms of
smaller parts (the way a sound wave can be analyzed in terms of
molecules). but i still can't 'see' EM waves. Can anybody help?
Thanks!
AH
It is admirable of you to wish to understand. It would be most helpful
if you were to let us know what background you have in physics and
mathematics. What do you know about electricity, magnetism and electric
circuits?

In essence, changing electric fields generate changing magnetic fields.
These changing magnetic fields generate changing electric fields. Each
of these fields generates the other and by interacting allows
propagation of the energy in these fields.

In a sound wave, you have a similar interaction. A pressure wave
generates up a velocity wave which in turn generates the pressure wave
again.

There are many other examples where two entities generate a wave. A
steady wind causes slight ripples that deflect the wind in an up and
down direction. This in turn accentuates ripple size.

Bill
--
An old man would be better off never having been born.
Szczepan Bialek
2010-05-31 17:47:04 UTC
Permalink
Post by akh2103
Hi--
I'm a recent humanities grad trying to get a grasp of EM. I can
understand/visualize sound waves propagating through a medium. But I
can't understand or visualize EM waves in the same way.
Read Maxwell: http://en.wikisource.org/wiki/On_Physical_Lines_of_Force
Post by akh2103
I get that EM
waves are 'fundamental' and can't be broken down/explained in terms of
smaller parts
In Maxwell's model are the two substances: massive, compressible magnetic
substance and massles, incompressible electricity.
Maxwell's model of eather was stripped away by Royal Society in 1864.
Post by akh2103
the way a sound wave can be analyzed in terms of
molecules). but i still can't 'see' EM waves. Can anybody help?
Royal Society had the excelent intuition.
No magnetic substance were discowered but the electrons.
The electrons are massive and the electron gas is compressible. In such gas
the electric waves propagate.

In textbooks are the all theories: "like sound" (the oldest), EM and
photons.

So you can analize the electric waves in terms of electrons (in place of
molecules).
S*
Post by akh2103
Thanks!
AH
slawek
2010-05-31 19:33:12 UTC
Permalink
Post by Szczepan Bialek
No magnetic substance were discowered but the electrons.
The electrons are massive and the electron gas is compressible. In such
gas the electric waves propagate.
Very nice, but what about quarks? You misses all hadrons, Sir.
Post by Szczepan Bialek
So you can analize the electric waves in terms of electrons (in place of
molecules).
Yes, but the wave "exist" even in the empty space. We do not write about
Alfven waves anyway...
Szczepan Bia³ek
2010-06-01 08:10:35 UTC
Permalink
Post by slawek
Post by Szczepan Bialek
No magnetic substance were discowered but the electrons.
The electrons are massive and the electron gas is compressible. In such
gas the electric waves propagate.
Very nice, but what about quarks? You misses all hadrons, Sir.
Post by Szczepan Bialek
So you can analize the electric waves in terms of electrons (in place of
molecules).
Yes, but the wave "exist" even in the empty space. We do not write about
Alfven waves anyway...
The empty space is in schools only. "Though the space between the stars is
emptier than the best vacuums created on the Earth (those are enclosed
spaces devoid of matter, not the household cleaning appliances), there is
some material between the stars composed of gas and dust. This material is
called the interstellar medium (ISM)". From:
http://www.astronomynotes.com/ismnotes/s2.htm

In the ISM are also the electrons.
Free electrons are in conductors. The speed of the electric waves are the
same in conductors and the space.
S*
slawek
2010-06-01 18:14:31 UTC
Permalink
Post by Szczepan Bia³ek
The empty space is in schools only. "Though the space between the stars is
So you missing your drill, mabye.
blackhead
2010-06-01 21:49:10 UTC
Permalink
Post by slawek
Post by Szczepan Bia³ek
The empty space is in schools only. "Though the space between the stars is
So you missing your drill, mabye.
He quoted from the link
slawek
2010-06-01 21:58:31 UTC
Permalink
Post by blackhead
He quoted from the link
False statements remain false when cited.
blackhead
2010-06-02 21:22:28 UTC
Permalink
Post by slawek
Post by blackhead
He quoted from the link
False statements remain false when cited.
He stated "The empty space is in schools only" and then gave the link,
implying there's little intelligence there, in his opinion.

You implied he had a screw loose, when you typed: "so you missing your
drill maybe". It looks as if you have made a mistake in thinking he
shares the view contained in he link - he doesn't.

Larry
Bill Miller
2010-06-04 16:32:40 UTC
Permalink
Post by akh2103
Hi--
I'm a recent humanities grad trying to get a grasp of EM. I can
understand/visualize sound waves propagating through a medium. But I
can't understand or visualize EM waves in the same way. I get that EM
waves are 'fundamental' and can't be broken down/explained in terms of
smaller parts (the way a sound wave can be analyzed in terms of
molecules). but i still can't 'see' EM waves. Can anybody help?
Thanks!
AH
After looking at some of the replies, I suspect you may still be having
dificulties!

Sound waves are transmitted mechanically through a medium of some sort. A
simple and reasonably accurate explanation of the process is that a molecule
is mechanically disturbed by the sound source. The molecule then bumps into
another molecule and another. If there is no medium, (as in a vacuum) there
is no sound.

EM is different.

Chances are you have run a plastic comb through your hair, waved it over
little pieces of paper and seen them move. The paper was influenced by the
Electric (E) Field associated with the electric charges on the comb. You
would see the same effect in the vacuum of space.

Chances are you have waved a permanent mgnet over pieces of iron filings and
seen them move. The filings were influenced by the Magnetic (M) Field
associated with the magnet. You would see the same effect in the vacuum of
space.

In both of these experiments, as you move the comb or magnet away, the
effect diminishes and ultimately becomes unmeasurable.

But something that seems "magical" can occur when Electric and Magnetic
fields combine. The combined effect can be measured at interstellar
distances!

The "why" is way too complex for an email. But we can talk a bit about how
it originated and what's going on.

In the mid-to-late 1800s, scientists, experimenters and mathematicians were
attempting to develop a set of mathematical formulas that would accurately
describe their measurements of Electrical and Magnetic phenomena.

James Clerk Maxwell derived a set of equations that now bear his name:
Maxwell's Equations. One form of these equations is called "Differential
Equations."

Differential Equations had been around for quite a while, and mathematicians
had developed general solutions for many of them -- including the forms
discovered by Maxwell.

One solution of Maxwells Equations leads to what is called a "Wave
Equation," A wave equation describes the characteristics of a sinusoidal
wave. 19th century scientists quickly realized that, according to Maxwell,
it *should* be possible to generate en EM Wave that could "fly" through
space.

Within a few years, Heinrich Hertz was able to generate those waves. And
that led to radio, TV, cell phones, GPS, Radar etc. etc.

Since numerous mechanical devices also generate sinusoidal waves, it is
tempting to look at how these devices operate and then attempt to apply
those principles to EM.

Here is a simple example: a pendulum. When we lift the free end, we impart
Potential Energy (PE) to it. When we release it, the pendulum will swing
downward until it reaches the bottom. At that time, all the PE has been
changed to Kinetic Energy (KE) and the arm will swing upward. At the top of
the swing (disregarding friction and air resistance) the pendulum will stop,
reverse its path and repeat the process "forever."

Thus, the energy goes from PE to KE to PE... etc.

This type of motion is called harmonic motion and is described by a
sinusoidal wave.

Since an EM wave is composed of an E component and an M component, it is
tempting to jump to the conclusion that there is an energy exchange from E
to M to E...etc.

But there is a *huge* difference between the two processes In the case of
the pendulum (and other similar mechanical harmonic devices) the PE and KE
are out of phase. That means that when the PE is at a maximum amplitude, the
KE is zero, and vice-versa.

In the case of the EM wave, when the E is maximum, so is the M. When the E
is zero, so is the M. Put another way, E cannot *cause* M and vice versa.

The actual cause of E and M is electrical charges and their motion. An EM
wave is called a "transverse" wave. Sound waves, etc are "longitudinal"
waves. They are very different.

Confusing? Yep. Don't feel badly if you have difficulty. A lot of really
smart and well educated folks are in the same boat!

I hope this helps! Good luck in learning more about this very fascinating
subject!

Bill Miller
glen herrmannsfeldt
2010-06-04 17:57:29 UTC
Permalink
Post by Bill Miller
Post by akh2103
I'm a recent humanities grad trying to get a grasp of EM. I can
understand/visualize sound waves propagating through a medium. But I
can't understand or visualize EM waves in the same way. I get that EM
waves are 'fundamental' and can't be broken down/explained in terms of
smaller parts (the way a sound wave can be analyzed in terms of
molecules). but i still can't 'see' EM waves. Can anybody help?
After looking at some of the replies, I suspect you may still be having
dificulties!
Sound waves are transmitted mechanically through a medium of some sort. A
simple and reasonably accurate explanation of the process is that a molecule
is mechanically disturbed by the sound source. The molecule then bumps into
another molecule and another. If there is no medium, (as in a vacuum) there
is no sound.
(big snip)

This is true, but does the bumping molecule approach really help
understand a sound wave? It is most obvious in terms of pressure
and density, which can be explained in terms of molecules, but once
you write it in terms of pressure and density it is just about
as abstract as E and M.

Anyway, Feynman has a good explanation of this in one of his books.

Or rather a good explanation of how hard it is to explain.

-- glen
Szczepan Bia³ek
2010-06-04 18:23:56 UTC
Permalink
Post by glen herrmannsfeldt
Post by Bill Miller
Sound waves are transmitted mechanically through a medium of some sort. A
simple and reasonably accurate explanation of the process is that a molecule
is mechanically disturbed by the sound source. The molecule then bumps into
another molecule and another. If there is no medium, (as in a vacuum) there
is no sound.
(big snip)
This is true, but does the bumping molecule approach really help
understand a sound wave?
It is possible to do the mechanical model. It is the best approach.
Post by glen herrmannsfeldt
It is most obvious in terms of pressure
and density, which can be explained in terms of molecules,
Bumping molecules.
Post by glen herrmannsfeldt
but once
you write it in terms of pressure and density it is just about
as abstract as E and M.
So the bumping is the best.
Post by glen herrmannsfeldt
Anyway, Feynman has a good explanation of this in one of his books.
Or rather a good explanation of how hard it is to explain.
So Bill is better than Feymman.
S*
Bill Miller
2010-06-04 18:33:51 UTC
Permalink
Post by glen herrmannsfeldt
Post by Bill Miller
Sound waves are transmitted mechanically through a medium of some sort. A
simple and reasonably accurate explanation of the process is that a molecule
is mechanically disturbed by the sound source. The molecule then bumps into
another molecule and another. If there is no medium, (as in a vacuum) there
is no sound.
(big snip)
This is true, but does the bumping molecule approach really help
understand a sound wave? It is most obvious in terms of pressure
and density, which can be explained in terms of molecules, but once
you write it in terms of pressure and density it is just about
as abstract as E and M.
Anyway, Feynman has a good explanation of this in one of his books.
Or rather a good explanation of how hard it is to explain.
-- glen
An alternate explanation might have been the analogy of the executive desk
thingy with the balls. You drop a ball and the energy bumps from ball to
ball and causes the last one to jump. This shows clearly the energy
transfer. But it misses the mark in explaining waves.

If Feynman had trouble explaining this, then I don't feel so bad!

Bill
Szczepan Bia³ek
2010-06-05 07:39:57 UTC
Permalink
Post by Bill Miller
An alternate explanation might have been the analogy of the executive desk
thingy with the balls. You drop a ball and the energy bumps from ball to
ball and causes the last one to jump. This shows clearly the energy
transfer. But it misses the mark in explaining waves.
One pulse makes the soliton.

If the first ball is mechanically disturbed by the sound source with the
steady frequency we have the wave.
But the first ball needs the time to come back to the sound source. It is
the reason that the upper limit of the frequency exists.
Post by Bill Miller
If Feynman had trouble explaining this, then I don't feel so bad!
Your "bumping waves" are the right way.
Many prefer the acting for distance but also many prefer acting only by the
contacts (bumping).
S*
Szczepan Bia³ek
2010-06-04 18:13:02 UTC
Permalink
Post by Bill Miller
After looking at some of the replies, I suspect you may still be having
dificulties!
Sound waves are transmitted mechanically through a medium of some sort. A
simple and reasonably accurate explanation of the process is that a
molecule is mechanically disturbed by the sound source. The molecule then
bumps into another molecule and another. If there is no medium, (as in a
vacuum) there is no sound.
In the wire are free electrons. Is possible that the electron is
mechanically disturbed by the source. The electron then bumps into
another electron and another.
Post by Bill Miller
EM is different.
Chances are you have run a plastic comb through your hair, waved it over
little pieces of paper and seen them move. The paper was influenced by the
Electric (E) Field associated with the electric charges on the comb. You
would see the same effect in the vacuum of space.
But now NASA says that in the space are the free electrons.
Post by Bill Miller
Chances are you have waved a permanent mgnet over pieces of iron filings
and seen them move. The filings were influenced by the Magnetic (M) Field
associated with the magnet. You would see the same effect in the vacuum of
space.
The "magnetc field" is produced by moving electric charges. It is also
electric field.
Post by Bill Miller
In both of these experiments, as you move the comb or magnet away, the
effect diminishes and ultimately becomes unmeasurable.
But something that seems "magical" can occur when Electric and Magnetic
fields combine. The combined effect can be measured at interstellar
distances!
The "why" is way too complex for an email. But we can talk a bit about how
it originated and what's going on.
In the mid-to-late 1800s, scientists, experimenters and mathematicians
were attempting to develop a set of mathematical formulas that would
accurately describe their measurements of Electrical and Magnetic
phenomena.
Maxwell's Equations. One form of these equations is called "Differential
Equations."
Differential Equations had been around for quite a while, and
mathematicians had developed general solutions for many of them --
including the forms discovered by Maxwell.
One solution of Maxwells Equations leads to what is called a "Wave
Equation," A wave equation describes the characteristics of a sinusoidal
wave. 19th century scientists quickly realized that, according to Maxwell,
it *should* be possible to generate en EM Wave that could "fly" through
space.
Within a few years, Heinrich Hertz was able to generate those waves. And
that led to radio, TV, cell phones, GPS, Radar etc. etc.
Since numerous mechanical devices also generate sinusoidal waves, it is
tempting to look at how these devices operate and then attempt to apply
those principles to EM.
Here is a simple example: a pendulum. When we lift the free end, we impart
Potential Energy (PE) to it. When we release it, the pendulum will swing
downward until it reaches the bottom. At that time, all the PE has been
changed to Kinetic Energy (KE) and the arm will swing upward. At the top
of the swing (disregarding friction and air resistance) the pendulum will
stop, reverse its path and repeat the process "forever."
Thus, the energy goes from PE to KE to PE... etc.
This type of motion is called harmonic motion and is described by a
sinusoidal wave.
Since an EM wave is composed of an E component and an M component, it is
tempting to jump to the conclusion that there is an energy exchange from E
to M to E...etc.
But there is a *huge* difference between the two processes In the case of
the pendulum (and other similar mechanical harmonic devices) the PE and KE
are out of phase. That means that when the PE is at a maximum amplitude,
the KE is zero, and vice-versa.
In the case of the EM wave, when the E is maximum, so is the M. When the E
is zero, so is the M. Put another way, E cannot *cause* M and vice versa.
The both are the same.
Post by Bill Miller
The actual cause of E and M is electrical charges and their motion. An EM
wave is called a "transverse" wave. Sound waves, etc are "longitudinal"
waves. They are very different.
The math by Maxwell and Heaviside is for the transversal.
For the longitudinal (The electron then bumps into another electron and
another.) the math is in the acoustics.

In the textbooks are the all theories. "Like sound", EM and photons.
Why do you prefer the EM?
Post by Bill Miller
Confusing? Yep. Don't feel badly if you have difficulty. A lot of really
smart and well educated folks are in the same boat!
The Royal Society stripped away the EM in 1864. Only Kelvin (a friend of
Maxwell) said "I do not understand your ideas".
Post by Bill Miller
I hope this helps! Good luck in learning more about this very fascinating
subject!
Maxwll's papers are in the excellent English.
S*
Bill Miller
2010-06-04 22:05:11 UTC
Permalink
"Szczepan Bia�ek" <***@wp.pl> wrote in message news:4c094235$0$2598$***@news.neostrada.pl...
<snip>
Post by Szczepan Bia³ek
But now NASA says that in the space are the free electrons.
S... the last time you said something outrageous like this, it had to do
with Acid Rain and Fly Ash. I thought you were crazy. Then I looked at the
website you referred to (in Hungarian, Czech or ???) ran it through the
Google translator and sure enough. You were -- in my opinion -- correct.

So... what is a link or source for this, please? Roughly how many electrons
per cubic anything??
Post by Szczepan Bia³ek
Post by Bill Miller
But something that seems "magical" can occur when >>
<snip>
Post by Szczepan Bia³ek
Post by Bill Miller
In the case of the EM wave, when the E is maximum, so is the M. When the
E is zero, so is the M. Put another way, E cannot *cause* M and vice
versa.
The both are the same.
Not so sure. There actually seem to be different *types* of E field (and M
field?) that interact with other items in different ways depending on
whether there is relative physical motion between them. So, in that sense
they are the "same." I'm still trying to pretend like I'm understanding all
this!.
Post by Szczepan Bia³ek
Post by Bill Miller
The actual cause of E and M is electrical charges and their motion. An EM
wave is called a "transverse" wave. Sound waves, etc are "longitudinal"
waves. They are very different.
The math by Maxwell and Heaviside is for the transversal.
For the longitudinal (The electron then bumps into another electron and
another.) the math is in the acoustics.
If, in fact there are sufficient quantity of electrons in the space occupied
by an EM wave, then yes it is possible for a longitudinal wave to exist.
(This has been recognized for quite awhile in plasmas. So I guess the
question must be: "When does a plasma stop acting as a plasma and start
acting like a vacuum?) In a "pure" vacuum, longitududinal waves are
nonexistent. (Or at least I have seen several proofs of their absence and
none for their presence. Also no measurements AFAIK.) Longitudinal waves
also can appear as a kind of "ground effect" at very low frequencies.
Post by Szczepan Bia³ek
In the textbooks are the all theories. "Like sound", EM and photons.
Why do you prefer the EM?
Post by Bill Miller
Confusing? Yep. Don't feel badly if you have difficulty. A lot of really
smart and well educated folks are in the same boat!
The Royal Society stripped away the EM in 1864. Only Kelvin (a friend of
Maxwell) said "I do not understand your ideas".
Marconi believed in Transverse waves only. Tesla supported the Longitudinal
Wave concept. (my note... Tesla seems to have worked almost exclusively at
Very Low Frequencies so he may have been the first to not this VLF
longitudinal effect.) Lord Kelvin apparently "mediated" a meeting between
Marconi and Tesla to decide who was right. Apparently, Lord Kelvin, on
exiting the meeting, was asked, "who was right?" His cryptic response was
that "They both are." I have never seen any clarification of this statement.

All the best,

Bill
Post by Szczepan Bia³ek
Post by Bill Miller
I hope this helps! Good luck in learning more about this very fascinating
subject!
Maxwll's papers are in the excellent English.
S*
Szczepan Bia³ek
2010-06-05 08:14:49 UTC
Permalink
Post by Bill Miller
<snip>
Post by Szczepan Bia³ek
But now NASA says that in the space are the free electrons.
S... the last time you said something outrageous like this, it had to do
with Acid Rain and Fly Ash. I thought you were crazy. Then I looked at the
website you referred to (in Hungarian, Czech or ???) ran it through the
Google translator and sure enough. You were -- in my opinion -- correct.
I citate carefully selected links only.
Post by Bill Miller
So... what is a link or source for this, please? Roughly how many
electrons per cubic anything??
Now are the physics of the dense plasma and the rare plasma (space plasma).
There are more equations than the four.

For it are many publications like this:

""space that is devoid of electrons" is only in schools."
"In space "About 99% of the interstellar medium is gas with about 90% of it
in the form of hydrogen (atomic or molecular form), 10% helium, and traces
of other elements" From: http://www.astronomynotes.com/ismnotes/s3.htm

"Some elements are ionized. So there are the electrons"

"How many electrons per cubic anything?"
"It can means the level and fluctuations. Now
everybody measure it:
http://www.academicjournals.org/ijlis/PDF/pdf2009/July/Okeke%20et%20al.pdf
Post by Bill Miller
Post by Szczepan Bia³ek
The both are the same.
Not so sure. There actually seem to be different *types* of E field (and M
field?) that interact with other items in different ways depending on
whether there is relative physical motion between them. So, in that sense
they are the "same." I'm still trying to pretend like I'm understanding
all this!.
Post by Szczepan Bia³ek
Post by Bill Miller
The actual cause of E and M is electrical charges and their motion. An
EM wave is called a "transverse" wave. Sound waves, etc are
"longitudinal" waves. They are very different.
The math by Maxwell and Heaviside is for the transversal.
For the longitudinal (The electron then bumps into another electron and
another.) the math is in the acoustics.
If, in fact there are sufficient quantity of electrons in the space
occupied by an EM wave, then yes it is possible for a longitudinal wave to
exist. (This has been recognized for quite awhile in plasmas. So I guess
the question must be: "When does a plasma stop acting as a plasma and
start acting like a vacuum?)
In the vacum are ions, electron and dust.

Ions are the medium for acoustic waves and the electrons are for the
electric waves.
The same like in wire. In the wire travel the sound waves and the electric
waves. With the differen speeds, of courese.
Post by Bill Miller
In a "pure" vacuum, longitududinal waves are nonexistent.
No pure vacum. Everywhere is the interstellar medium (ISM).

(Or at least I have seen several proofs of their absence and
Post by Bill Miller
none for their presence. Also no measurements AFAIK.) Longitudinal waves
also can appear as a kind of "ground effect" at very low frequencies.
Post by Szczepan Bia³ek
In the textbooks are the all theories. "Like sound", EM and photons.
Why do you prefer the EM?
Post by Bill Miller
Confusing? Yep. Don't feel badly if you have difficulty. A lot of really
smart and well educated folks are in the same boat!
The Royal Society stripped away the EM in 1864. Only Kelvin (a friend of
Maxwell) said "I do not understand your ideas".
Marconi believed in Transverse waves only. Tesla supported the
Longitudinal Wave concept. (my note... Tesla seems to have worked almost
exclusively at Very Low Frequencies so he may have been the first to not
this VLF longitudinal effect.) Lord Kelvin apparently "mediated" a meeting
between Marconi and Tesla to decide who was right. Apparently, Lord
Kelvin, on exiting the meeting, was asked, "who was right?" His cryptic
response was that "They both are." I have never seen any clarification of
this statement.
It was in the Stokes time.
Even now for somebody the water waves are transversal.
But they are mainly longitudinal. This and the mass transport and the energy
transport was describesd by Stokes before 1850:
http://en.wikipedia.org/wiki/Stokes_drift
All real waves have the both components.

All the best,
S*
Bill Miller
2010-06-08 16:43:54 UTC
Permalink
Post by Szczepan Bia³ek
Post by Bill Miller
<snip>
Post by Szczepan Bia³ek
But now NASA says that in the space are the free electrons.
S... the last time you said something outrageous like this, it had to do
with Acid Rain and Fly Ash. I thought you were crazy. Then I looked at
the website you referred to (in Hungarian, Czech or ???) ran it through
the Google translator and sure enough. You were -- in my opinion --
correct.
I citate carefully selected links only.
Post by Bill Miller
So... what is a link or source for this, please? Roughly how many
electrons per cubic anything??
Now are the physics of the dense plasma and the rare plasma (space
plasma). There are more equations than the four.
""space that is devoid of electrons" is only in schools."
"In space "About 99% of the interstellar medium is gas with about 90% of it
in the form of hydrogen (atomic or molecular form), 10% helium, and traces
of other elements" From: http://www.astronomynotes.com/ismnotes/s3.htm
"Some elements are ionized. So there are the electrons"
"How many electrons per cubic anything?"
"It can means the level and fluctuations. Now
http://www.academicjournals.org/ijlis/PDF/pdf2009/July/Okeke%20et%20al.pdf
Post by Bill Miller
Post by Szczepan Bia³ek
The both are the same.
Not so sure. There actually seem to be different *types* of E field (and
M field?) that interact with other items in different ways depending on
whether there is relative physical motion between them. So, in that sense
they are the "same." I'm still trying to pretend like I'm understanding
all this!.
Post by Szczepan Bia³ek
Post by Bill Miller
The actual cause of E and M is electrical charges and their motion. An
EM wave is called a "transverse" wave. Sound waves, etc are
"longitudinal" waves. They are very different.
The math by Maxwell and Heaviside is for the transversal.
For the longitudinal (The electron then bumps into another electron and
another.) the math is in the acoustics.
If, in fact there are sufficient quantity of electrons in the space
occupied by an EM wave, then yes it is possible for a longitudinal wave
to exist. (This has been recognized for quite awhile in plasmas. So I
guess the question must be: "When does a plasma stop acting as a plasma
and start acting like a vacuum?)
In the vacum are ions, electron and dust.
Ions are the medium for acoustic waves and the electrons are for the
electric waves.
The same like in wire. In the wire travel the sound waves and the electric
waves. With the differen speeds, of courese.
Post by Bill Miller
In a "pure" vacuum, longitududinal waves are nonexistent.
No pure vacum. Everywhere is the interstellar medium (ISM).
(Or at least I have seen several proofs of their absence and
Post by Bill Miller
none for their presence. Also no measurements AFAIK.) Longitudinal waves
also can appear as a kind of "ground effect" at very low frequencies.
Post by Szczepan Bia³ek
In the textbooks are the all theories. "Like sound", EM and photons.
Why do you prefer the EM?
Post by Bill Miller
Confusing? Yep. Don't feel badly if you have difficulty. A lot of
really smart and well educated folks are in the same boat!
The Royal Society stripped away the EM in 1864. Only Kelvin (a friend of
Maxwell) said "I do not understand your ideas".
Marconi believed in Transverse waves only. Tesla supported the
Longitudinal Wave concept. (my note... Tesla seems to have worked almost
exclusively at Very Low Frequencies so he may have been the first to not
this VLF longitudinal effect.) Lord Kelvin apparently "mediated" a
meeting between Marconi and Tesla to decide who was right. Apparently,
Lord Kelvin, on exiting the meeting, was asked, "who was right?" His
cryptic response was that "They both are." I have never seen any
clarification of this statement.
It was in the Stokes time.
Even now for somebody the water waves are transversal.
But they are mainly longitudinal. This and the mass transport and the
http://en.wikipedia.org/wiki/Stokes_drift
All real waves have the both components.
All the best,
S*
Well, it took me a while to see if I could quantify what level of ionization
is required in order to support a tramsverse EM wave.

Here is some data, courtesy ok Kirk McDonald at Princeton. Its sort of good
news/bad news in support of your position:

1. Longitudinal waves in plasmas have a frequency close to the plasma
frequency.

This means (if I did the math right) that the number density N is ~ 1 / r_0
lambda where R_0 = classical electron radius ~ 1e-15 m

So for, say, lambda ~ 1 m, we need N ~ 1e15 ions/m^3.or, N ~ 1e9 ions/em^3.

Air has ~ 3e19 molecules/cm^3.

So, fully ionizied air at a pressure 1e-10 bar could support the
longitudinal waves.

I gather that these conditions exist in the upper atmosphere, and it is
claimed that longitudinal waves have been detected (indirectly) there:

Kirk has a link to a paper that describes this effect:

http://puhep1.princeton.edu/~mcdonald/examples/plasma/kumar_jpd_40_3396_07.pdf

Apparently there are two different types of Longitudinal Waves that appeared
in this study. The "acoustic" is analagous to sound. The Bernstein waves are
also longitudinal but appear to require a static magnetic field in order to
form.

The Bernstein wave type is a fairly broadband phenomenon, but it is most
prominent at frequencies at/near the electron cyclotron frequency and that
is in the low MegaHertz range. I'm not sure about the bandwidth of acoustic
EM waves.

So... yes it seems that at least upper atmosphere longitudinal waves can
form, But it *appears* to me that at distances far removed from planetary
bodies, the plasma density is so low that lingitudinal wave formation would
be problematical, at best. In deep space, static magnetic fields would see m
to preclude the Bernstein waves. Also, this seems to be a low MHz
phenonmenon that may be of little interest except as a curiosity.

But I could be wrong!

All the best,

Bill
Szczepan Bia³ek
2010-06-09 07:55:44 UTC
Permalink
Post by Bill Miller
Post by Szczepan Bia³ek
In the vacum are ions, electron and dust.
Ions are the medium for acoustic waves and the electrons are for the
electric waves.
The same like in wire. In the wire travel the sound waves and the
electric waves. With the differen speeds, of courese.
Post by Bill Miller
In a "pure" vacuum, longitududinal waves are nonexistent.
No pure vacum. Everywhere is the interstellar medium (ISM).
Post by Bill Miller
Marconi believed in Transverse waves only. Tesla supported the
Longitudinal Wave concept. (my note... Tesla seems to have worked almost
exclusively at Very Low Frequencies so he may have been the first to not
this VLF longitudinal effect.) Lord Kelvin apparently "mediated" a
meeting between Marconi and Tesla to decide who was right. Apparently,
Lord Kelvin, on exiting the meeting, was asked, "who was right?" His
cryptic response was that "They both are." I have never seen any
clarification of this statement.
It was in the Stokes time.
Even now for somebody the water waves are transversal.
But they are mainly longitudinal. This and the mass transport and the
http://en.wikipedia.org/wiki/Stokes_drift
All real waves have the both components.
Well, it took me a while to see if I could quantify what level of
ionization is required in order to support a tramsverse EM wave.
Transverse or longitudinal?
Post by Bill Miller
Here is some data, courtesy ok Kirk McDonald at Princeton. Its sort of
1. Longitudinal waves in plasmas have a frequency close to the plasma
frequency.
This means (if I did the math right) that the number density N is ~ 1 /
r_0 lambda where R_0 = classical electron radius ~ 1e-15 m
So for, say, lambda ~ 1 m, we need N ~ 1e15 ions/m^3.or, N ~ 1e9 ions/em^3.
Air has ~ 3e19 molecules/cm^3.
So, fully ionizied air at a pressure 1e-10 bar could support the
longitudinal waves.
I gather that these conditions exist in the upper atmosphere, and it is
http://puhep1.princeton.edu/~mcdonald/examples/plasma/kumar_jpd_40_3396_07.pdf
Apparently there are two different types of Longitudinal Waves that
appeared in this study. The "acoustic" is analagous to sound. The
Bernstein waves are also longitudinal but appear to require a static
magnetic field in order to form.
The Bernstein wave type is a fairly broadband phenomenon, but it is most
prominent at frequencies at/near the electron cyclotron frequency and that
is in the low MegaHertz range. I'm not sure about the bandwidth of
acoustic EM waves.
So... yes it seems that at least upper atmosphere longitudinal waves can
form, But it *appears* to me that at distances far removed from planetary
bodies, the plasma density is so low that lingitudinal wave formation
would be problematical, at best. In deep space, static magnetic fields
would see m to preclude the Bernstein waves. Also, this seems to be a low
MHz phenonmenon that may be of little interest except as a curiosity.
But I could be wrong!
Tesla was right:
"Nikola Tesla wrote: " I showed that the universal medium is a gaseous body
in which only longitudinal pulses can be propagated, involving alternating
compressions and expansions similar to those produced by sound waves in the
air. Thus, a wireless transmitter does not propagate Hertz waves, which are
a myth, but sound waves in the ether, behaving in every respect like those
in the air, except that, owing to the great elastic force and extremely
small density of the medium, their speed is that of light."

So "like sound" is also known theory.
For me it is the only sesible and experimentally proved. EM and the photons
not. But in schools must be the all.
S*
Bill Miller
2010-06-09 20:57:17 UTC
Permalink
"Szczepan Bia�ek" <***@wp.pl> wrote in message news:4c0f4902$0$17091$***@news.neostrada.pl...
<snip> Well, it took me a while to see if I could quantify what level of
Post by Szczepan Bia³ek
Post by Bill Miller
ionization is required in order to support a tramsverse EM wave.
Transverse or longitudinal?
Yes. My fingers were not properly connected to my brain. I meant
Longitudinal.
Post by Szczepan Bia³ek
"Nikola Tesla wrote: " I showed that the universal medium is a gaseous
body in which only longitudinal pulses can be propagated, involving
alternating compressions and expansions similar to those produced by sound
waves in the air. Thus, a wireless transmitter does not propagate Hertz
waves, which are a myth, but sound waves in the ether, behaving in every
respect like those in the air, except that, owing to the great elastic
force and extremely small density of the medium, their speed is that of
light."
So "like sound" is also known theory.
For me it is the only sesible and experimentally proved. EM and the
photons not. But in schools must be the all.
S*
Well, you are in good company with the likes of Tesla. Unfortunately, you
are very much in the minority with regard to the concept of ether. Although
Tesla's explanation quoted above is consistent with waves in a plasma, it is
not consistent with a whole host of experimental data.

As far as photons go, I'm afaraid I don't have even the foggiest notion of
what they are. I suspect I'm in the vast majority here, but few are willing
to say so!

Bill
Szczepan Bia³ek
2010-06-10 09:04:00 UTC
Permalink
Post by Bill Miller
<snip> Well, it took me a while to see if I could quantify what level of
Post by Szczepan Bia³ek
Post by Bill Miller
ionization is required in order to support a tramsverse EM wave.
Transverse or longitudinal?
Yes. My fingers were not properly connected to my brain. I meant
Longitudinal.
Post by Szczepan Bia³ek
"Nikola Tesla wrote: " I showed that the universal medium is a gaseous
body in which only longitudinal pulses can be propagated, involving
alternating compressions and expansions similar to those produced by
sound waves in the air. Thus, a wireless transmitter does not propagate
Hertz waves, which are a myth, but sound waves in the ether, behaving in
every respect like those in the air, except that, owing to the great
elastic force and extremely small density of the medium, their speed is
that of light."
So "like sound" is also known theory.
For me it is the only sesible and experimentally proved. EM and the
photons not. But in schools must be the all.
S*
Well, you are in good company with the likes of Tesla. Unfortunately, you
are very much in the minority with regard to the concept of ether.
I am thinking that everybody see the plasma in the space.
Post by Bill Miller
Although Tesla's explanation quoted above is consistent with waves in a
plasma, it is not consistent with a whole host of experimental data.
Transverse waves need the solid medium. Which the experimental data prove
it?
Post by Bill Miller
As far as photons go, I'm afaraid I don't have even the foggiest notion of
what they are. I suspect I'm in the vast majority here, but few are
willing to say so!
EM is for explanation of the light polarisation and the photon for the
photoelectric effect.
Now are the better explanations.
S*
p***@ic.ac.uk
2010-06-08 16:48:41 UTC
Permalink
Post by Bill Miller
In the case of the EM wave, when the E is maximum, so is the M. When the E
is zero, so is the M. Put another way, E cannot *cause* M and vice versa.
But I _can_ say that a magnetic field that varies in time demands that
there are corresponding spatial variations in the electric field, and
vice versa -- this is what is specified by the current-free versions
of the curl Maxwell's equations.

Thus presumably the linkage between a given set of freely propagating
fields _is_ causal, even if not being as simple as you might like.

I can, after all, simulate freely propagating EM fields (with an
accuracy limited only by the numerics) using FDTD, an explicitly
causal algorithm for solving Maxwell's equations. Is it really
possible for a causal algorithm to accurately simulate a non-causal
process?
Post by Bill Miller
The actual cause of E and M is electrical charges and their motion. An EM
wave is called a "transverse" wave. Sound waves, etc are "longitudinal"
waves. They are very different.
They are indeed very different -- but there _are_ transverse "sound"
waves -- e.g. LO phonons in crystals.
--
---------------------------------+---------------------------------
Dr. Paul Kinsler
Blackett Laboratory (Photonics) (ph) +44-20-759-47734 (fax) 47714
Imperial College London, ***@physics.org
SW7 2AZ, United Kingdom. http://www.qols.ph.ic.ac.uk/~kinsle/
Bill Miller
2010-06-08 20:28:43 UTC
Permalink
Post by p***@ic.ac.uk
Post by Bill Miller
In the case of the EM wave, when the E is maximum, so is the M. When the E
is zero, so is the M. Put another way, E cannot *cause* M and vice versa.
But I _can_ say that a magnetic field that varies in time demands that
there are corresponding spatial variations in the electric field, and
vice versa -- this is what is specified by the current-free versions
of the curl Maxwell's equations.
This is descriptive rather than causal. Causality requires that the cause
precede the effect, If the *presumed* cause happens at the same time as the
*presumed* effect, then we can be certain that neither causes the other.
Using Occam's Razor, the simplest solution is that they are both being
caused by another, earlier event -- or set of events.

Please see Jefimenko and also Panofsky & Phillips for the actual causal
equations. The Wiki link,
http://en.wikipedia.org/wiki/Jefimenko%27s_equations provides some
additional information, various links and explanations.
Post by p***@ic.ac.uk
Thus presumably the linkage between a given set of freely propagating
fields _is_ causal, even if not being as simple as you might like.
I'm sorry, but there is no field linkage that causes one to cause the other
to cause.... etc.
Post by p***@ic.ac.uk
I can, after all, simulate freely propagating EM fields (with an
accuracy limited only by the numerics) using FDTD, an explicitly
causal algorithm for solving Maxwell's equations. Is it really
possible for a causal algorithm to accurately simulate a non-causal
process?
Well, it's been a while since I used FDTD software, but I use NEC (Numerical
Electromagnetic Code) very often to simulate antenna performance. This
software, the most advanced version of which is DOD controlled, is unable to
deal with very small dimensions (WRT wavelength) and cannot handle current
sources in small loop circuitry.

So, here is a challenge for you. Use your FDTD software to construct a
closely spaced Parallel Plate capacitor. Apply AC to it and then use the
FDTD to "measure" the magnetic field between the plates that is supposedly
caused by the dD/dt componet in Maxwell.

If you succeed in getting a reading, then you will have accomplished what
has not been done in over 100 years using *real* capacitors.Either there are
a lot of really incompetent experimental physicists running around or... the
field does not exist.
Post by p***@ic.ac.uk
Post by Bill Miller
The actual cause of E and M is electrical charges and their motion. An EM
wave is called a "transverse" wave. Sound waves, etc are "longitudinal"
waves. They are very different.
They are indeed very different -- but there _are_ transverse "sound"
waves -- e.g. LO phonons in crystals.
Yes. There are also Longitudinal EM waves found in Plasmas (see another post
to S for the link) and at Very Low Frequncies in a sort of "quasi-ground
effect" mode whose characteristics were measured around a century ago. (Or
more IF this is the mode that Tesla was working with.)

All the best,

Bill Miller
Post by p***@ic.ac.uk
--
---------------------------------+---------------------------------
Dr. Paul Kinsler
Blackett Laboratory (Photonics) (ph) +44-20-759-47734 (fax) 47714
SW7 2AZ, United Kingdom. http://www.qols.ph.ic.ac.uk/~kinsle/
Benj
2010-06-09 06:13:56 UTC
Permalink
Post by Bill Miller
This is descriptive rather than causal. Causality requires that the cause
precede the effect, If the *presumed* cause happens at the same time as the
*presumed* effect, then we can be certain that neither causes the other.
Using Occam's Razor, the simplest solution is that they are both being
caused by another, earlier event -- or set of events.
Please see Jefimenko and also Panofsky & Phillips for the actual causal
equations. The Wiki link,http://en.wikipedia.org/wiki/Jefimenko%27s_equationsprovides some
additional information, various links and explanations.
Whoa there Bill! Asserting that electric and magnetic fields don't
"cause each other"? You are pushing awfully close to joining me in
"official crankdom"! Pretty soon they'll be accusing you of "making
all this crap up" as they do me in spite of providing the references!
Post by Bill Miller
If you succeed in getting a reading, then you will have accomplished what
has not been done in over 100 years using *real* capacitors.Either there are
a lot of really incompetent experimental physicists running around or... the
field does not exist.
And you are a "displacement current" denier as well? Oh My! Your
kookosity rating is definitely on the rise!

Seriously, though, isn't it interesting and a statement on present day
science that if we take one of the most successful theories (Maxwell's
Equations) that have been successfully applied in the development of a
myriad of fantastic devices, and start to look closely at it, the
standard statements taught in freshman textbooks suddenly start to
fall apart and a lot of holes start to appear.

My explanation for that is that when humans have the choice of doing
something right or taking the quick and easy route, most take the easy
way. Is that some kind of corollary to Occam's Razor?

"There is always an easy solution to every human problem... neat,
plausible and wrong!"
H.L. Mencken

"Everything should be made as simple as possible, but not simpler!"
A. Einstein
Bill Miller
2010-06-09 21:50:00 UTC
Permalink
"Benj" <***@iwaynet.net> wrote in message news:aeb7bfd3-5f7f-48a5-b3d9-***@k39g2000yqd.googlegroups.com...
<snip>
My explanation for that is that when humans have the choice of doing
something right or taking the quick and easy route, most take the easy
way. Is that some kind of corollary to Occam's Razor?

According to a contemporary sketch, William of Ockham (Occam) was, in fact,
clean shaven. So I guess he practised what he preached.
Szczepan Bia³ek
2010-06-09 07:41:12 UTC
Permalink
Post by Bill Miller
Post by p***@ic.ac.uk
But I _can_ say that a magnetic field that varies in time demands that
there are corresponding spatial variations in the electric field, and
vice versa -- this is what is specified by the current-free versions
of the curl Maxwell's equations.
This is descriptive rather than causal. Causality requires that the cause
precede the effect, If the *presumed* cause happens at the same time as
the *presumed* effect, then we can be certain that neither causes the
other.
Maxwell also did not know:
"We have as yet given no answers to the questions, " How are these vortices
set in rotation?" and "Why are they arranged according to the known laws of
lines of force about magnets and currents?" These questions are certainly of
a higher order of difficulty than either of the former ; and I wish to
separate the suggestions I may offer by way of provisional answer to them,
from the mechanical deductions which resolved the first question, and the
hypothesis of vortices which gave a probable answer to the second.
We have, in fact, now come to inquire into the physical con- nexion of these
vortices with electric currents, while we are still in doubt as to the
nature of electricity, whether it is one sub- stance, two substances, or not
a substance at all, or in what way it differs from matter, and how it is
connected with it. " From:
http://en.wikisource.org/wiki/On_Physical_Lines_of_Force

Nobody can know how to do the magnetic whirl. Everybody can assume that such
exsists. But for what?

S*
John Polasek
2010-06-10 04:07:08 UTC
Permalink
Post by Bill Miller
Post by p***@ic.ac.uk
Post by Bill Miller
In the case of the EM wave, when the E is maximum, so is the M. When the E
is zero, so is the M. Put another way, E cannot *cause* M and vice versa.
But I _can_ say that a magnetic field that varies in time demands that
there are corresponding spatial variations in the electric field, and
vice versa -- this is what is specified by the current-free versions
of the curl Maxwell's equations.
This is descriptive rather than causal. Causality requires that the cause
precede the effect, If the *presumed* cause happens at the same time as the
*presumed* effect, then we can be certain that neither causes the other.
???We can be certain? that is an unsupported presumption.
Post by Bill Miller
Using Occam's Razor, the simplest solution is that they are both being
caused by another, earlier event -- or set of events.
So if I send a current from a current source through a resistor and a
voltage develops then the current must precede the voltage, in order
to be the cause??
Or, from a low impedance source I place a voltage across the resistor
and a current flows, then the voltage must precede the current in
order to be the cause??
Or it's possible that they are simultaneous and that the causative
agent is then only identified by whether it's a voltage or current
source.
Why are they simultaneous? because the voltage and current are
mutually coupled by a common resistance that they share.

And so it is in space where the fields are joined by the 377 ohms of
free space: H = E/Z. Z = sqrt(mu/eps) =377 ohms.
Post by Bill Miller
Please see Jefimenko and also Panofsky & Phillips for the actual causal
equations. The Wiki link,
http://en.wikipedia.org/wiki/Jefimenko%27s_equations provides some
additional information, various links and explanations.
You carefully point to Jefimenko, but I think you'll agree that those
equations are much too abstruse to convey useful information.

And as far as Panofsky including those equations in his 1962 edition
(Mine is earlier), please take note that the authors P&P have no idea
even of how a simple vacuum capacitor works:
" two plates bearing charges of +Q and -Q" and
"inserting dielectrics taking care not to touch the plates with it
...the forces are reduced".

They have no idea why the forces between them would be reduced "since
the dielectric did not touch the charges"! Nowhere in my copy do they
take a battery and connect it to the capacitor so as to demonstrate
how it is the dielectric and the vacuum dielectrics that is stressed
by the field E. There is no mention of a battery or other source of
EMF anywhere in the book.
They are still 100 years behind, obsessed with the charges as from an
electrophorus residing on the plates and exerting forces between them.
By the way, where they get exactly +Q to match -Q?
They had no idea about permittivity nor permeability to the extent
that they actually believed in them or had any feel for them, while
Impelled by pressure to incorporate them after the SI system was
adopted.
Post by Bill Miller
Post by p***@ic.ac.uk
Thus presumably the linkage between a given set of freely propagating
fields _is_ causal, even if not being as simple as you might like.
I'm sorry, but there is no field linkage that causes one to cause the other
to cause.... etc.
Post by p***@ic.ac.uk
I can, after all, simulate freely propagating EM fields (with an
accuracy limited only by the numerics) using FDTD, an explicitly
causal algorithm for solving Maxwell's equations. Is it really
possible for a causal algorithm to accurately simulate a non-causal
process?
Well, it's been a while since I used FDTD software, but I use NEC (Numerical
Electromagnetic Code) very often to simulate antenna performance. This
software, the most advanced version of which is DOD controlled, is unable to
deal with very small dimensions (WRT wavelength) and cannot handle current
sources in small loop circuitry.
So, here is a challenge for you. Use your FDTD software to construct a
closely spaced Parallel Plate capacitor. Apply AC to it and then use the
FDTD to "measure" the magnetic field between the plates that is supposedly
caused by the dD/dt componet in Maxwell.
If you succeed in getting a reading, then you will have accomplished what
has not been done in over 100 years using *real* capacitors.Either there are
a lot of really incompetent experimental physicists running around or... the
field does not exist.
Post by p***@ic.ac.uk
Post by Bill Miller
The actual cause of E and M is electrical charges and their motion. An EM
wave is called a "transverse" wave. Sound waves, etc are "longitudinal"
waves. They are very different.
They are indeed very different -- but there _are_ transverse "sound"
waves -- e.g. LO phonons in crystals.
Yes. There are also Longitudinal EM waves found in Plasmas (see another post
to S for the link) and at Very Low Frequncies in a sort of "quasi-ground
effect" mode whose characteristics were measured around a century ago. (Or
more IF this is the mode that Tesla was working with.)
All the best,
Bill Miller
Post by p***@ic.ac.uk
--
---------------------------------+---------------------------------
Dr. Paul Kinsler
Blackett Laboratory (Photonics) (ph) +44-20-759-47734 (fax) 47714
SW7 2AZ, United Kingdom. http://www.qols.ph.ic.ac.uk/~kinsle/
John Polasek
Bill Miller
2010-06-10 19:36:51 UTC
Permalink
<snip>
Post by John Polasek
So if I send a current from a current source through a resistor and a
voltage develops then the current must precede the voltage, in order
to be the cause??
Or, from a low impedance source I place a voltage across the resistor
and a current flows, then the voltage must precede the current in
order to be the cause??
Or it's possible that they are simultaneous and that the causative
agent is then only identified by whether it's a voltage or current
source.
Why are they simultaneous? because the voltage and current are
mutually coupled by a common resistance that they share.
And so it is in space where the fields are joined by the 377 ohms of
free space: H = E/Z. Z = sqrt(mu/eps) =377 ohms.
I'm glad you made this point. In your voltage analogy, R is a variable and
in a general case, may be complex. In the case of free space, Z0 is a
constant and *is not/cannot be* complex. This is important because, as you
correctly noted, H/E = Z. If E and H are not time synchronous (ie in phase)
then this ratio will be complex. But the only way that E could cause H that
causes E etc. is if the two waves are out of phase.

I realize this was not your point, but I couldn't resist expanding on your
example to show the logical impossibility presented by others.
Post by John Polasek
Post by Bill Miller
Please see Jefimenko and also Panofsky & Phillips for the actual causal
equations. The Wiki link,
http://en.wikipedia.org/wiki/Jefimenko%27s_equations provides some
additional information, various links and explanations.
You carefully point to Jefimenko, but I think you'll agree that those
equations are much too abstruse to convey useful information.
Quite to the contrary. A careful analysis shows that the E field -- as
explained by Jefimenko -- contains both static and kinetic components. In
particular, the kinetic E field, called Ek, exerts a "dragging" force on
nearby objects. Ek explains why Lentz's law works as it does. It is not
possible to derive (or explain) Lentz's law in terms of Maxwell's Equations.
It is only when they are expandd to their causal roots that we see what is
going on.
Post by John Polasek
And as far as Panofsky including those equations in his 1962 edition
(Mine is earlier), please take note that the authors P&P have no idea
" two plates bearing charges of +Q and -Q" and
"inserting dielectrics taking care not to touch the plates with it
...the forces are reduced".
They have no idea why the forces between them would be reduced "since
the dielectric did not touch the charges"! Nowhere in my copy do they
take a battery and connect it to the capacitor so as to demonstrate
how it is the dielectric and the vacuum dielectrics that is stressed
by the field E. There is no mention of a battery or other source of
EMF anywhere in the book.
They are still 100 years behind, obsessed with the charges as from an
electrophorus residing on the plates and exerting forces between them.
By the way, where they get exactly +Q to match -Q?
They had no idea about permittivity nor permeability to the extent
that they actually believed in them or had any feel for them, while
Impelled by pressure to incorporate them after the SI system was
adopted.
I'm not going to touch this. The possibility that a quoted source *may*
contain errors that are totally unrelated to the matter at hand is
interesting but irrelevant. This especially true in my citation, since P&P
and Jefimenko used entirely different concepts and methods to come up with
identical results. That is generally considered to be extremely strong
evidence that the results are correct.
<snip>

All the best,

Bill
glen herrmannsfeldt
2010-06-10 20:50:36 UTC
Permalink
(snip)
Post by Bill Miller
Post by John Polasek
And so it is in space where the fields are joined by the 377 ohms of
free space: H = E/Z. Z = sqrt(mu/eps) =377 ohms.
I'm glad you made this point. In your voltage analogy, R is a variable and
in a general case, may be complex. In the case of free space, Z0 is a
constant and *is not/cannot be* complex. This is important because, as you
correctly noted, H/E = Z. If E and H are not time synchronous (ie in phase)
then this ratio will be complex. But the only way that E could cause H that
causes E etc. is if the two waves are out of phase.
I do remember wondering about this in my E&M class many years ago.

In the usual diagram of a wave moving in the Z direction, with
E in the X direction and B in the Y direction, the E and B fields
are in phase. That is, the zero crossings are at the same position
in space. That is, a diagram of the wave frozen in time.

Now, if one considers a water wave with height being potential
energy and velocity (of the water) being kinetic energy, the
two are out of phase with each other.

If you consider, for example, the E field being potential energy,
and B (related to dE/dt) being kinetic energy, it would again
seem that they should be out of phase.

I don't remember actually going through it in detail enough
to agree or disagree with the diagram.

-- glen
Bill Miller
2010-06-10 21:22:00 UTC
Permalink
Post by glen herrmannsfeldt
(snip)
Post by Bill Miller
Post by John Polasek
And so it is in space where the fields are joined by the 377 ohms of
free space: H = E/Z. Z = sqrt(mu/eps) =377 ohms.
I'm glad you made this point. In your voltage analogy, R is a variable and
in a general case, may be complex. In the case of free space, Z0 is a
constant and *is not/cannot be* complex. This is important because, as you
correctly noted, H/E = Z. If E and H are not time synchronous (ie in phase)
then this ratio will be complex. But the only way that E could cause H that
causes E etc. is if the two waves are out of phase.
I do remember wondering about this in my E&M class many years ago.
In the usual diagram of a wave moving in the Z direction, with
E in the X direction and B in the Y direction, the E and B fields
are in phase. That is, the zero crossings are at the same position
in space. That is, a diagram of the wave frozen in time.
Now, if one considers a water wave with height being potential
energy and velocity (of the water) being kinetic energy, the
two are out of phase with each other.
If you consider, for example, the E field being potential energy,
and B (related to dE/dt) being kinetic energy, it would again
seem that they should be out of phase.
I don't remember actually going through it in detail enough
to agree or disagree with the diagram.
-- glen
YES! (Excuse the "shout" please. To repeat a phrase from the "A Team,"

"I Love it when a plan comes together!"

You just put it together.

All the best,

Bill
John Polasek
2010-06-10 22:21:19 UTC
Permalink
Post by Bill Miller
<snip>
Post by John Polasek
So if I send a current from a current source through a resistor and a
voltage develops then the current must precede the voltage, in order
to be the cause??
Or, from a low impedance source I place a voltage across the resistor
and a current flows, then the voltage must precede the current in
order to be the cause??
Or it's possible that they are simultaneous and that the causative
agent is then only identified by whether it's a voltage or current
source.
Why are they simultaneous? because the voltage and current are
mutually coupled by a common resistance that they share.
And so it is in space where the fields are joined by the 377 ohms of
free space: H = E/Z. Z = sqrt(mu/eps) =377 ohms.
I'm glad you made this point. In your voltage analogy, R is a variable and
in a general case, may be complex. In the case of free space, Z0 is a
constant and *is not/cannot be* complex. This is important because, as you
correctly noted, H/E = Z. If E and H are not time synchronous (ie in phase)
then this ratio will be complex. But the only way that E could cause H that
causes E etc. is if the two waves are out of phase.
JP: Yes, they are 180° out of phase just as in a lumped constant LC
parallel circuit, where the voltage across each one is equal and
opposite and the potential summed around the loop is zero. They trade
potential and kinetic energies back and forth once they get started.
Ldi/dt + 1/C int idt = 0
if one of these is a sine wave, the other is a minus sine wave, being
two derivatives apart.
Z0 is indeed complex-and I don't mean mathematically. I have tried to
discover a tensor for it considering the complexity of
curl E = -mu*dH/dt
so E and H are connected by much more than a coupling constant Z.
Post by Bill Miller
I realize this was not your point, but I couldn't resist expanding on your
example to show the logical impossibility presented by others.
Post by John Polasek
Post by Bill Miller
Please see Jefimenko and also Panofsky & Phillips for the actual causal
equations. The Wiki link,
http://en.wikipedia.org/wiki/Jefimenko%27s_equations provides some
additional information, various links and explanations.
You carefully point to Jefimenko, but I think you'll agree that those
equations are much too abstruse to convey useful information.
Quite to the contrary. A careful analysis shows that the E field -- as
explained by Jefimenko -- contains both static and kinetic components. In
particular, the kinetic E field, called Ek, exerts a "dragging" force on
nearby objects. Ek explains why Lentz's law works as it does. It is not
possible to derive (or explain) Lentz's law in terms of Maxwell's Equations.
It is only when they are expandd to their causal roots that we see what is
going on.
JP: Are you sure you're talking about Lentzs law? it only says that a
current generates a magnetic field that provides a negative back
current opposing the original-nothing abstruse.
Neither does classical E&M have V = Ldi/dt nor Ndphi/dt-I guess only
engineers are so privileged.
As I mentioned, Panofsky does not use volts- just charges-which sounds
like Jefimenko (and also Bill Miller).
What is your version of the constitutive equations? Mine is
D = epsE and B = muH.
It requires SI units to make sense of anything. Vacuum with its
permittivity *acts* as if it had charges in the form of charge pairs.
We've had this discussion before.

Just a couple of weeks ago I saw an article by a woman physicist who
put her faith in Jackson and who wanted to transmit separately the B
wave and the E wave to see what would happen. That's what comes of the
agnostic CGS philosophy. Poppycock.
Post by Bill Miller
Post by John Polasek
And as far as Panofsky including those equations in his 1962 edition
(Mine is earlier), please take note that the authors P&P have no idea
" two plates bearing charges of +Q and -Q" and
"inserting dielectrics taking care not to touch the plates with it
...the forces are reduced".
They have no idea why the forces between them would be reduced "since
the dielectric did not touch the charges"! Nowhere in my copy do they
take a battery and connect it to the capacitor so as to demonstrate
how it is the dielectric and the vacuum dielectrics that is stressed
by the field E. There is no mention of a battery or other source of
EMF anywhere in the book.
They are still 100 years behind, obsessed with the charges as from an
electrophorus residing on the plates and exerting forces between them.
By the way, where they get exactly +Q to match -Q?
They had no idea about permittivity nor permeability to the extent
that they actually believed in them or had any feel for them, while
Impelled by pressure to incorporate them after the SI system was
adopted.
I'm not going to touch this. The possibility that a quoted source *may*
contain errors that are totally unrelated to the matter at hand is
interesting but irrelevant. This especially true in my citation, since P&P
and Jefimenko used entirely different concepts and methods to come up with
identical results. That is generally considered to be extremely strong
evidence that the results are correct.
<snip>
All the best,
Bill
John Polasek
Benj
2010-06-12 09:47:58 UTC
Permalink
Post by Bill Miller
But the only way that E could cause H that
Post by Bill Miller
causes E etc. is if the two waves are out of phase.
JP:  Yes, they are 180 out of phase just as in a lumped constant LC
parallel circuit, where the voltage across each one is equal and
opposite and the potential summed  around the loop is zero. They trade
potential and kinetic energies back and forth once they get started.
        Ldi/dt + 1/C int idt = 0
if one of these is a sine wave, the other is a minus sine wave, being
two derivatives apart.
Z0 is indeed complex-and I don't mean mathematically. I have tried to
discover a tensor for it considering  the complexity of
                curl E = -mu*dH/dt
so E and H are connected by much more than a coupling constant Z.
"There is always an easy solution to every human problem - neat,
plausible, and wrong!"

H.L. Mencken
Bill Miller
2010-06-14 16:25:45 UTC
Permalink
<snip>> And so it is in space where the fields are joined by the 377 ohms of
Post by John Polasek
Post by Bill Miller
Post by John Polasek
free space: H = E/Z. Z = sqrt(mu/eps) =377 ohms.
Yes. See below.
Post by John Polasek
Post by Bill Miller
I'm glad you made this point. In your voltage analogy, R is a variable and
in a general case, may be complex. In the case of free space, Z0 is a
constant and *is not/cannot be* complex. This is important because, as you
correctly noted, H/E = Z. If E and H are not time synchronous (ie in phase)
then this ratio will be complex. But the only way that E could cause H that
causes E etc. is if the two waves are out of phase.
JP: Yes, they are 180° out of phase just as in a lumped constant LC
parallel circuit, where the voltage across each one is equal and
opposite and the potential summed around the loop is zero. They trade
potential and kinetic energies back and forth once they get started.
Ldi/dt + 1/C int idt = 0
if one of these is a sine wave, the other is a minus sine wave, being
two derivatives apart.
Z0 is indeed complex-and I don't mean mathematically. I have tried to
discover a tensor for it considering the complexity of
curl E = -mu*dH/dt
so E and H are connected by much more than a coupling constant Z.
John, if E and H are complex numbers, how can their ratio be a *real*
number?
Post by John Polasek
Post by Bill Miller
I realize this was not your point, but I couldn't resist expanding on your
example to show the logical impossibility presented by others.
Post by John Polasek
Post by John Polasek
JP: Are you sure you're talking about Lentzs law? it only says that a
current generates a magnetic field that provides a negative back
current opposing the original-nothing abstruse.
You claimed that an anlyisis of Jefimenko/ P&P's equations would yield no
useful information. I showed that it provides at least one piece of useful
information that is not found in Maxwell.
Post by John Polasek
Neither does classical E&M have V = Ldi/dt nor Ndphi/dt-I guess only
engineers are so privileged.
Yep. These equations work. They are just not causal.
Post by John Polasek
As I mentioned, Panofsky does not use volts- just charges-which sounds
like Jefimenko (and also Bill Miller).
What is your version of the constitutive equations? Mine is
D = epsE and B = muH.
It requires SI units to make sense of anything. Vacuum with its
permittivity *acts* as if it had charges in the form of charge pairs.
We've had this discussion before.
Yes. I think I'll pass on having it again.
Post by John Polasek
Just a couple of weeks ago I saw an article by a woman physicist who
put her faith in Jackson and who wanted to transmit separately the B
wave and the E wave to see what would happen. That's what comes of the
agnostic CGS philosophy. Poppycock.
Actually, this line of thinking has cost a lot of people a lot of money.
Devices like the CFA, EH and a host of others have attempted to "synthesize"
an EM wave by generating separate E and H fields and then combining them.
So, I agree with your conclusion, if not the cause: Poppycock.

<snip>

All the best,

Bill Miller
Benj
2010-06-12 09:37:47 UTC
Permalink
Post by John Polasek
Post by Bill Miller
Please see Jefimenko and also Panofsky & Phillips for the actual causal
equations. The Wiki link,
http://en.wikipedia.org/wiki/Jefimenko%27s_equationsprovides some
additional information, various links and explanations.
You carefully point to Jefimenko, but I think you'll agree that those
equations are much too abstruse to convey useful information.
Sorry we DON'T agree. In fact you really don't have to actually
understand the derivation of them and the mathematics to easily see a
number of extremely important facts.

The first of these can be seen by looking at the causal equation for
the H (magnetic) field. That field is caused by only two sources.
There are the retarded current density [J] and the time rate of change
of the retarded current density d[J]/dt. Note that these are the ONLY
source of H! That means that H is a result of CURRENT and it's changes
and NOTHING else. The widely held belief that a changing electric
field causes a magnetic field (displacement current) is clearly bogus
right from Maxwell's theory! The usual "proof" provided of
displacement current is in error because the equations used in the
proof are not causal. This is completely easy to understand without
any understanding of the mathematics involved. (The only question that
would remain would be are Jefimenko's derivations correct. I say they
are and I've not see anybody refute them or his assumptions to get
them.)

Similarly if we look at the causal equations for an E field we see
that it has only three sources. Those are retarded charge, the time
rate of change of retarded charge and as above the time rate of change
of retarded current. The latter is especially interesting in that it
represents the phenomena known as induction. Again it is widely
believed that a changing magnetic field induces an E field (EMF).
Clearly a totally bogus idea! The ONLY source for induced E is d[J]/
dt! Hence it is a current that causes the magnetic field AND causes
the induction! Faraday's Law is just something handy to use but does
not indicate what is going on.

And all this is obvious with ZERO understanding of the actual
mathematics just by looking at what variables are involved.
And I call this "useful information".

Of course, going to Physics Forums and explaining these facts will
quickly get you banned for life as a "crackpot", so I wouldn't
recommend doing that.
John Polasek
2010-06-12 13:50:42 UTC
Permalink
Post by Benj
Post by John Polasek
Post by Bill Miller
Please see Jefimenko and also Panofsky & Phillips for the actual causal
equations. The Wiki link,
http://en.wikipedia.org/wiki/Jefimenko%27s_equationsprovides some
additional information, various links and explanations.
You carefully point to Jefimenko, but I think you'll agree that those
equations are much too abstruse to convey useful information.
Sorry we DON'T agree. In fact you really don't have to actually
understand the derivation of them and the mathematics to easily see a
number of extremely important facts.
The first of these can be seen by looking at the causal equation for
the H (magnetic) field. That field is caused by only two sources.
There are the retarded current density [J] and the time rate of change
of the retarded current density d[J]/dt. Note that these are the ONLY
source of H! That means that H is a result of CURRENT and it's changes
and NOTHING else. The widely held belief that a changing electric
field causes a magnetic field (displacement current) is clearly bogus
right from Maxwell's theory! The usual "proof" provided of
displacement current is in error because the equations used in the
proof are not causal. This is completely easy to understand without
any understanding of the mathematics involved. (The only question that
would remain would be are Jefimenko's derivations correct. I say they
are and I've not see anybody refute them or his assumptions to get
them.)
Similarly if we look at the causal equations for an E field we see
that it has only three sources. Those are retarded charge, the time
rate of change of retarded charge and as above the time rate of change
of retarded current. The latter is especially interesting in that it
represents the phenomena known as induction. Again it is widely
believed that a changing magnetic field induces an E field (EMF).
Clearly a totally bogus idea! The ONLY source for induced E is d[J]/
dt! Hence it is a current that causes the magnetic field AND causes
the induction! Faraday's Law is just something handy to use but does
not indicate what is going on.
And all this is obvious with ZERO understanding of the actual
mathematics just by looking at what variables are involved.
And I call this "useful information".
Of course, going to Physics Forums and explaining these facts will
quickly get you banned for life as a "crackpot", so I wouldn't
recommend doing that.
Just briefly, you really couldn't have Poynting's vector, the product
of E and H, if they were entirely independent of each other. It would
just be symbology, like D = E and B = H.
Does he have a book?
John Polasek
Benj
2010-06-14 01:57:58 UTC
Permalink
Post by John Polasek
Just briefly, you really couldn't have Poynting's vector, the product
of E and H, if they were entirely independent of each other. It would
just be symbology, like D = E and B = H.
E and H are not independent of each other! As you can easily see from
the causal equations BOTH are caused by a current! Hence current is
the "cause" and both E and H propagate outward from there. For
example. There is a mathematical theorem that allows one to replace
radiation upon an aperture with a "current" source on the same
aperture. Say you had an open waveguide. One can replace the open end
of the waveguide with an aperture with an "equivalent" current source
on it. That current source can then be used to calculate the far field
pattern of that "antenna" by a relatively simple transform. So dig
this. If one has radiation (say light) incident upon some apertures
(say two slits of some given width) One can actually create an
equivalent current source at those apertures and use that source with
a transform (Fourier in this case) to actually calculate the TRUE
measured diffraction pattern... NOT the overly simplified "Young's"
pattern that physicists always use! [Assumes point or line sources]

This is all great stuff but mostly ignored except by people needing
real answers to real problems!
Post by John Polasek
Does he have a book?
Several of them. Search his name on Amazon.com. The one of most
interest in this question is "causality, Electromagnetic induction and
Gravitation". His E&M textbook though old is VERY interesting in that
he worked out quite a few examples that other texts don't! PS. his
books are not expensive and really worth it though as you have noted
he does tend to get very mathematical at times!
John Polasek
2010-06-13 20:00:35 UTC
Permalink
Post by Benj
Post by John Polasek
Post by Bill Miller
Please see Jefimenko and also Panofsky & Phillips for the actual causal
equations. The Wiki link,
http://en.wikipedia.org/wiki/Jefimenko%27s_equationsprovides some
additional information, various links and explanations.
You carefully point to Jefimenko, but I think you'll agree that those
equations are much too abstruse to convey useful information.
Sorry we DON'T agree. In fact you really don't have to actually
understand the derivation of them and the mathematics to easily see a
number of extremely important facts.
The first of these can be seen by looking at the causal equation for
the H (magnetic) field. That field is caused by only two sources.
There are the retarded current density [J] and the time rate of change
of the retarded current density d[J]/dt. Note that these are the ONLY
source of H! That means that H is a result of CURRENT and it's changes
and NOTHING else. The widely held belief that a changing electric
field causes a magnetic field (displacement current) is clearly bogus
right from Maxwell's theory! The usual "proof" provided of
displacement current is in error because the equations used in the
proof are not causal. This is completely easy to understand without
any understanding of the mathematics involved. (The only question that
would remain would be are Jefimenko's derivations correct. I say they
are and I've not see anybody refute them or his assumptions to get
them.)
Similarly if we look at the causal equations for an E field we see
that it has only three sources. Those are retarded charge, the time
rate of change of retarded charge and as above the time rate of change
of retarded current. The latter is especially interesting in that it
represents the phenomena known as induction. Again it is widely
believed that a changing magnetic field induces an E field (EMF).
Clearly a totally bogus idea! The ONLY source for induced E is d[J]/
dt!
Faraday's law says V = -Ndphi/dt where phi = integral B.dA.
dphi/dt is the only thing that can cause the secondary in a
transformer to have a voltage proportional to the number of its turns.
The primary and secondary are related by the turns ratio. Voltage
makes current makes flux makes voltage makes current (if the secondary
has a load).
In the face of this, your assertion is reduced merely to a recitation.
Post by Benj
Hence it is a current that causes the magnetic field AND causes
the induction! Faraday's Law is just something handy to use but does
not indicate what is going on.
This simple example of transformer action is a perfect model to
demonstrate what is going on.
In an open circuited secondary, the voltage increases from one end to
the other of the winding. (my equations):
V = integral E.dL
E = dV/dL
I can just see (in my minds eye), the vector E lying coiled around
with each turn of the wire, being the solenoidal field whose curl is
proportional to the axial time rate of change of integral B.dA.
curl E = -dB/dt
Yes, that seems to work! E and B work arm in arm and Faraday's law
makes complete sense, bringing Maxwell's equation to the laboratory.
snip
John Polasek
Bill Miller
2010-06-14 16:41:42 UTC
Permalink
On Sat, 12 Jun 2010 02:37:47 -0700 (PDT), Benj <snip>
Post by Benj
Hence it is a current that causes the magnetic field AND causes
the induction! Faraday's Law is just something handy to use but does
not indicate what is going on.
This simple example of transformer action is a perfect model to
demonstrate what is going on.
In an open circuited secondary, the voltage increases from one end to
V = integral E.dL
E = dV/dL
I can just see (in my minds eye), the vector E lying coiled around
with each turn of the wire, being the solenoidal field whose curl is
proportional to the axial time rate of change of integral B.dA.
curl E = -dB/dt
Yes, that seems to work! E and B work arm in arm and Faraday's law
makes complete sense, bringing Maxwell's equation to the laboratory.
snip
John Polasek
Jefimenko's textbook, Electricity and Magnetism has *many* examples
demonstrating the classical formulas such as Faraday's Law.
In his "Causality" book, he takes quite a number of those equations and
shows that using the causal equations leads to the same results such as L
di/dt etc.

The important thing is understanding that equivalent results does not
necessarily equate with causality. If it did, then someone over the last
century plus would have measured the magnetic field between the plates of a
capacitor that is "caused" by Displacement Current. It "ain't" there!

Bill Miller
Szczepan Bia³ek
2010-06-14 17:23:14 UTC
Permalink
Post by Bill Miller
The important thing is understanding that equivalent results does not
necessarily equate with causality. If it did, then someone over the last
century plus would have measured the magnetic field between the plates of
a capacitor that is "caused" by Displacement Current. It "ain't" there!
The displacement current is only in the oscillating form. No measuring
devices for such.

In electrolyte is simillar current but in DC form. There is a magnetic
field.
S*
Bill Miller
2010-06-14 18:45:11 UTC
Permalink
Post by Szczepan Bia³ek
Post by Bill Miller
The important thing is understanding that equivalent results does not
necessarily equate with causality. If it did, then someone over the last
century plus would have measured the magnetic field between the plates of
a capacitor that is "caused" by Displacement Current. It "ain't" there!
The displacement current is only in the oscillating form. No measuring
devices for such.
In electrolyte is simillar current but in DC form. There is a magnetic
field.
Yes. But this is a conduction current in which real electrons flow. The term
"Displacement Current" is an unfortunate one, since it does not represent a
real current and does not have a magnetic field that surrounds it (like in a
real cuurent.)
Post by Szczepan Bia³ek
S*
All the best,

Bill>
Szczepan Bia³ek
2010-06-15 08:00:39 UTC
Permalink
Post by Bill Miller
Post by Szczepan Bia³ek
Post by Bill Miller
The important thing is understanding that equivalent results does not
necessarily equate with causality. If it did, then someone over the last
century plus would have measured the magnetic field between the plates
of a capacitor that is "caused" by Displacement Current. It "ain't"
there!
The displacement current is only in the oscillating form. No measuring
devices for such.
In electrolyte is simillar current but in DC form. There is a magnetic
field.
Yes. But this is a conduction current in which real electrons flow. The
term "Displacement Current" is an unfortunate one, since it does not
represent a real current and does not have a magnetic field that surrounds
it (like in a real cuurent.)
It is a real AC current (John explained why it is).

The problem with EM is that the displacement current in the dielectric is
synchronous with the current in the adjacent wire.

The displacement current in the dielectric is incompressible (the number of
charges and the distances between them are constant).

But Maxwell assumed that the current in the wire also. It is known Maxwell's
mistake.

At the and of the wire (before the dielectric) the electrons are compressed
(the voltage is doubled). The doubled voltage produces the Tesla waves.

Analyse of EM is a lost of time.
Maxwell was a math genius and his math for the rotational vibrations in the
solid media are in use and are tought.
S*
John Polasek
2010-06-15 00:22:44 UTC
Permalink
Post by Bill Miller
On Sat, 12 Jun 2010 02:37:47 -0700 (PDT), Benj <snip>
Post by Benj
Hence it is a current that causes the magnetic field AND causes
the induction! Faraday's Law is just something handy to use but does
not indicate what is going on.
This simple example of transformer action is a perfect model to
demonstrate what is going on.
In an open circuited secondary, the voltage increases from one end to
V = integral E.dL
E = dV/dL
I can just see (in my minds eye), the vector E lying coiled around
with each turn of the wire, being the solenoidal field whose curl is
proportional to the axial time rate of change of integral B.dA.
curl E = -dB/dt
Yes, that seems to work! E and B work arm in arm and Faraday's law
makes complete sense, bringing Maxwell's equation to the laboratory.
snip
John Polasek
Jefimenko's textbook, Electricity and Magnetism has *many* examples
demonstrating the classical formulas such as Faraday's Law.
In his "Causality" book, he takes quite a number of those equations and
shows that using the causal equations leads to the same results such as L
di/dt etc.
The important thing is understanding that equivalent results does not
necessarily equate with causality. If it did, then someone over the last
century plus would have measured the magnetic field between the plates of a
capacitor that is "caused" by Displacement Current. It "ain't" there!
Bill Miller
This debate could go on forever. The conclusions would depend entirely
on whether you believe in a real permittivity and permeability. I must
conclude that you are of a CGS persuasion and have no use for those
concepts. Then there is no use proceeding.
About all those charges that cause everything, where are you getting
them and why hasn't it been noticed before?
As for displacement current, you need to keep in mind that it's not a
current (excluding AC applications which we are not discussing). It is
a momentary pulse. When the battery is connected to a capacitor there
is a transient pulse of current, typically of microsecond duration
determined by RC. Think of the practical difficulties in detecting
the attendant microsecond-long magnetic field.
Where are the experiments that show that there is no such 'current'?
During those microseconds the neutral dielectric bends a little and
the electrons peek out a little to produce D coul/m2. That is the
*effect* of the electric field. Analyze the equation del.D = rho.
After you disconnect the battery you will still read e.g. 100 volts,
because now this polarization changes roles and becomes the *cause*
and by itself produces 100 V e.g.

I get the impression you don't put much stock in Faraday's law either
But I just showed you a concrete example that is an exact application.
It would be useful if you could point to just one surprising new fact
evinced by your Jif man.
John Polasek
Bill Miller
2010-06-15 14:24:14 UTC
Permalink
Post by John Polasek
This debate could go on forever. The conclusions would depend entirely
on whether you believe in a real permittivity and permeability.
No. The conclusion depends on whether you believe real measurements. No
matter how delightful a theory may be, it must ultimately be subjected to
the scrutiny of real world measurements.

And the magnetic field "caused by" or "associated with" the displacement
current between the plates of a capacitor *has never been measured.*

.I must
Post by John Polasek
conclude that you are of a CGS persuasion and have no use for those
concepts. Then there is no use proceeding.
Then why are you doing so?
Post by John Polasek
About all those charges that cause everything, where are you getting
them and why hasn't it been noticed before?
C'mon John. Do I now understand that you do not believe in charges?
Post by John Polasek
As for displacement current, you need to keep in mind that it's not a
current (excluding AC applications which we are not discussing).
You may not be discussing AC applications. I certainly am.
Post by John Polasek
.It
What is "it?"
Post by John Polasek
a momentary pulse. When the battery is connected to a capacitor there
is a transient pulse of current, typically of microsecond duration
determined by RC. Think of the practical difficulties in detecting
the attendant microsecond-long magnetic field.
Where are the experiments that show that there is no such 'current'?
Actually, what you learned about capacitors is also not quite right. A
capacitor is nothing but a very short, very closely spaced transmission
line. The charging occurs in discrete steps. The smoothed form of the
charging curve is dependent on the ratio of the source resistance to the
characteristic impedance Zo of the line. It is usually exponential since R
is usually>>> Zo. But When R,Zo a capacitor can actually charge (briefly) to
a value that is higher than the source voltage. And if R=Zo, the capacitor
charges almost instantly to exactly 100% of the source voltage.

The above is OT since it has little to do with EM. (But a lot to do with EE)
Post by John Polasek
During those microseconds the neutral dielectric bends a little and
the electrons peek out a little to produce D coul/m2. That is the
*effect* of the electric field. Analyze the equation del.D = rho.
After you disconnect the battery you will still read e.g. 100 volts,
because now this polarization changes roles and becomes the *cause*
and by itself produces 100 V e.g.
I get the impression you don't put much stock in Faraday's law either
But I just showed you a concrete example that is an exact application.
It would be useful if you could point to just one surprising new fact
evinced by your Jif man.
I would say that there are two surprising facts. E does not cause H and H
does not cause E. Benj did a great job earlier in spelling out the analysis
of Jefimenko's equations.

Perhaps if you were to read his "Causality" book, you might see what we are
talking about. I am sure it is awfully difficult to make a valid judgment
based on snippets of information in a newsgroup.Read it, follow the logic
and the derivation. Then make your own determination.

All the best,

Bill Miller
Post by John Polasek
John Polasek
John Polasek
2010-06-15 15:46:59 UTC
Permalink
Post by Bill Miller
Post by John Polasek
This debate could go on forever. The conclusions would depend entirely
on whether you believe in a real permittivity and permeability.
No. The conclusion depends on whether you believe real measurements. No
matter how delightful a theory may be, it must ultimately be subjected to
the scrutiny of real world measurements.
And the magnetic field "caused by" or "associated with" the displacement
current between the plates of a capacitor *has never been measured.*
.I must
Post by John Polasek
conclude that you are of a CGS persuasion and have no use for those
concepts. Then there is no use proceeding.
Then why are you doing so?
Post by John Polasek
About all those charges that cause everything, where are you getting
them and why hasn't it been noticed before?
C'mon John. Do I now understand that you do not believe in charges?
Post by John Polasek
As for displacement current, you need to keep in mind that it's not a
current (excluding AC applications which we are not discussing).
You may not be discussing AC applications. I certainly am.
Even there it is not a current. The alternating electric field causes
a microscopic displacement of the elastically restrained charges,
merely oscillating minutely about their home positions. In a field of
100,000V/m I calculate 1.227e-27 m (vacuum).
Post by Bill Miller
Post by John Polasek
.It
What is "it?"
Post by John Polasek
a momentary pulse. When the battery is connected to a capacitor there
is a transient pulse of current, typically of microsecond duration
determined by RC. Think of the practical difficulties in detecting
the attendant microsecond-long magnetic field.
Where are the experiments that show that there is no such 'current'?
Actually, what you learned about capacitors is also not quite right. A
capacitor is nothing but a very short, very closely spaced transmission
line.
See my reply regarding the grounding problems in the OP's hobby
chassis. The earth has a self capacitance of 790 µF. What kind of a
transmission line is that? You really should stick to first principles
in order to be convincing, not lean on your expertise in transmission
lines.
Post by Bill Miller
The charging occurs in discrete steps. The smoothed form of the
charging curve is dependent on the ratio of the source resistance to the
characteristic impedance Zo of the line. It is usually exponential since R
is usually>>> Zo. But When R,Zo a capacitor can actually charge (briefly) to
a value that is higher than the source voltage. And if R=Zo, the capacitor
charges almost instantly to exactly 100% of the source voltage.
The above is OT since it has little to do with EM. (But a lot to do with EE)
Post by John Polasek
During those microseconds the neutral dielectric bends a little and
the electrons peek out a little to produce D coul/m2. That is the
*effect* of the electric field. Analyze the equation del.D = rho.
After you disconnect the battery you will still read e.g. 100 volts,
because now this polarization changes roles and becomes the *cause*
and by itself produces 100 V e.g.
I get the impression you don't put much stock in Faraday's law either
But I just showed you a concrete example that is an exact application.
It would be useful if you could point to just one surprising new fact
evinced by your Jif man.
I would say that there are two surprising facts. E does not cause H and H
does not cause E.
I gave you a fine description of how in the secondary winding the
vector E lies along the turns thus becoming a curled field; curl E Is
a straight vector on the axis of the solenoid, equal to -mu dH/dt. E
and H work hand-in-hand.
Post by Bill Miller
Benj did a great job earlier in spelling out the analysis
of Jefimenko's equations.
Could you be more specific and post the date on which this great job
occurred, otherwise it's a tedious fishing trip to find it?
Post by Bill Miller
Perhaps if you were to read his "Causality" book, you might see what we are
talking about. I am sure it is awfully difficult to make a valid judgment
based on snippets of information in a newsgroup.Read it, follow the logic
and the derivation. Then make your own determination.
All the best,
Bill Miller
Post by John Polasek
John Polasek
Bill Miller
2010-06-15 21:48:11 UTC
Permalink
"John Polasek" <***@cfl.rr.com> wrote in message news:***@4ax.com...
<snip>
Post by John Polasek
Post by Bill Miller
Actually, what you learned about capacitors is also not quite right. A
capacitor is nothing but a very short, very closely spaced transmission
line.
See my reply regarding the grounding problems in the OP's hobby
chassis. The earth has a self capacitance of 790 µF. What kind of a
transmission line is that? You really should stick to first principles
in order to be convincing, not lean on your expertise in transmission
lines.
Let's take it a step at a time. First, let's model the earth as a little
ball inside a big ball. Apply a step function in series with a finite
resistor.between the little ball and the big ball. Next watch what happens
over tim. The charge will build up in spep function fashion. It is a
complicated result but consistent with transmission line concepts (I never
said it was faster or easier. I just say it is correct.) BTW, the idea of
self-capacitance has always fascinated me. It comes about by using the
formula of a spherical capacitor when the inner diameter is the size of the
Earth and we allow the outer diameter to go to infinity. But it is not all
that useful since at infinity it takes an infinite amount of time to change
the charge on the inner ball.

But perhaps there is such a thing. In Zen Buddhism, the koan, "We know the
sound of two hands clapping. What is the sound of *one* hand clapping?" is
one of the first koans to be learned. Perhaps in Zen EM the equivalent koan
would be: "We know the capacitance of a two element capacitor. What is the
capacitance of a *one* element capacitor?" Ommmmmmm....
Post by John Polasek
Could you be more specific and post the date on which this great job
occurred, otherwise it's a tedious fishing trip to find it?
Glad to. June 12. It was in this thread in response to your postulate (based
only on information from Wiki and the thread) that Jefimenko's equations
provide no new or useful information. Reread Benj's explanation, look at the
equations and you will see that -- contrary to what is taught in every EE
and Electricity course in the world -- induction is not a phenomenon by
which a changing magnetic field causes an electric field to appear. It is a
nice happenstance that the equations that describe changing magnetism as
causing a changing voltage. The clue here is that the reverse has *never*
been shown to be true.

Then please recall that it is perfectly OK to use 22/7 to calculate the area
of a circle, even though 22/7 has nothing at all to do with pi.

All the best,

Bill
Post by John Polasek
Post by Bill Miller
Perhaps if you were to read his "Causality" book, you might see what we are
talking about. I am sure it is awfully difficult to make a valid judgment
based on snippets of information in a newsgroup.Read it, follow the logic
and the derivation. Then make your own determination.
All the best,
Bill Miller
Post by John Polasek
John Polasek
John Polasek
2010-06-16 00:58:39 UTC
Permalink
Post by Bill Miller
<snip>
Post by John Polasek
Post by Bill Miller
Actually, what you learned about capacitors is also not quite right. A
capacitor is nothing but a very short, very closely spaced transmission
line.
See my reply regarding the grounding problems in the OP's hobby
chassis. The earth has a self capacitance of 790 µF. What kind of a
transmission line is that? You really should stick to first principles
in order to be convincing, not lean on your expertise in transmission
lines.
Let's take it a step at a time. First, let's model the earth as a little
ball inside a big ball.
Why not try my method and then see if you want to go on with your
transmission line:
E = Q/4piepso*R^2
V = Q/4pieps0*R
C = Q/V = 4pieps0*R
Post by Bill Miller
Apply a step function in series with a finite
resistor.between the little ball and the big ball. Next watch what happens
over tim. The charge will build up in spep function fashion. It is a
complicated result but consistent with transmission line concepts (I never
said it was faster or easier. I just say it is correct.)
BTW, the idea of
self-capacitance has always fascinated me. It comes about by using the
formula of a spherical capacitor when the inner diameter is the size of couriers are
Earth and we allow the outer diameter to go to infinity. But it is not all
that useful since at infinity it takes an infinite amount of time to change
the charge on the inner ball.
It's a whole lot easier to specify charge Q, go out to radius R where
you can compute the field, (arrange an enclosing metal shell with
radius R) as I did above, and integrate the E field to get V, as I did
above, and the rest is left as an exercise for the student.
Post by Bill Miller
But perhaps there is such a thing. In Zen Buddhism, the koan, "We know the
sound of two hands clapping. What is the sound of *one* hand clapping?" is
one of the first koans to be learned. Perhaps in Zen EM the equivalent koan
would be: "We know the capacitance of a two element capacitor. What is the
capacitance of a *one* element capacitor?" Ommmmmmm....
Post by John Polasek
Could you be more specific and post the date on which this great job
occurred, otherwise it's a tedious fishing trip to find it?
Glad to. June 12. It was in this thread in response to your postulate (based
only on information from Wiki and the thread) that Jefimenko's equations
provide no new or useful information. Reread Benj's explanation, look at the
equations and you will see that -- contrary to what is taught in every EE
and Electricity course in the world -- induction is not a phenomenon by
which a changing magnetic field causes an electric field to appear. It is a
nice happenstance that the equations that describe changing magnetism as
causing a changing voltage. The clue here is that the reverse has *never*
been shown to be true.
"The first of these can be seen by looking at the causal equation for
the H (magnetic) field. That field is caused by only two sources.
There are the retarded current density [J] and the time rate of change
of the retarded current density d[J]/dt.
Note that these are the ONLY
source of H! That means that H is a result of CURRENT and it's changes
and NOTHING else. The widely held belief that a changing electric
field causes a magnetic field (displacement current) is clearly bogus
right from Maxwell's theory!"
Nonsense. The strength of H is defined by amp*turns/m. Wind a coil,
drive some current through it and there is your defined H. It's in
addition to your "only two sources, the retarded current density [J]
and the time rate of change of the retarded current density d[J]/dt".
Post by Bill Miller
Then please recall that it is perfectly OK to use 22/7 to calculate the area
of a circle, even though 22/7 has nothing at all to do with pi.
All the best,
Bill
Post by John Polasek
Post by Bill Miller
Perhaps if you were to read his "Causality" book, you might see what we are
talking about. I am sure it is awfully difficult to make a valid judgment
based on snippets of information in a newsgroup.Read it, follow the logic
and the derivation. Then make your own determination.
All the best,
Bill Miller
Post by John Polasek
John Polasek
John Polasek
Bill Miller
2010-06-16 13:26:37 UTC
Permalink
On Tue, 15 Jun 2010 17:48:11 -0400, "Bill Miller" <snip>
Post by Benj
Note that these are the ONLY
source of H! That means that H is a result of CURRENT and it's changes
and NOTHING else. The widely held belief that a changing electric
field causes a magnetic field (displacement current) is clearly bogus
right from Maxwell's theory!"
Nonsense. The strength of H is defined by amp*turns/m. Wind a coil,
drive some current through it and there is your defined H. It's in
addition to your "only two sources, the retarded current density [J]
and the time rate of change of the retarded current density d[J]/dt".
Maybe I'm the only one that thinks that Paul and Benj have just said the
same thing? And Paul disagrees with Benj?

Or perhaps this is another example of "Two people divided by the same
language."

All the best...

Bill
Bill Miller
2010-06-16 13:45:05 UTC
Permalink
Post by Bill Miller
On Tue, 15 Jun 2010 17:48:11 -0400, "Bill Miller" <snip>
Post by Benj
Note that these are the ONLY
source of H! That means that H is a result of CURRENT and it's changes
and NOTHING else. The widely held belief that a changing electric
field causes a magnetic field (displacement current) is clearly bogus
right from Maxwell's theory!"
Nonsense. The strength of H is defined by amp*turns/m. Wind a coil,
drive some current through it and there is your defined H. It's in
addition to your "only two sources, the retarded current density [J]
and the time rate of change of the retarded current density d[J]/dt".
Maybe I'm the only one that thinks that Paul and Benj have just said the
same thing? And Paul disagrees with Benj?
Or perhaps this is another example of "Two people divided by the same
language."
All the best...
Bill
Sorry! I missed my morning coffee. Please change "Paul" to "John." Benj
remains the same.
:-)
John Polasek
2010-06-16 15:00:06 UTC
Permalink
Post by Bill Miller
On Tue, 15 Jun 2010 17:48:11 -0400, "Bill Miller" <snip>
Post by Benj
Note that these are the ONLY
source of H! That means that H is a result of CURRENT and it's changes
and NOTHING else. The widely held belief that a changing electric
field causes a magnetic field (displacement current) is clearly bogus
right from Maxwell's theory!"
Nonsense. The strength of H is defined by amp*turns/m. Wind a coil,
drive some current through it and there is your defined H. It's in
addition to your "only two sources, the retarded current density [J]
and the time rate of change of the retarded current density d[J]/dt".
Maybe I'm the only one that thinks that Paul and Benj have just said the
same thing? And John disagrees with Benj?
Benj said:
I would say that there are two surprising facts. E does not cause H
and H does not cause E.
John replied:
I gave you a fine description of how in the secondary winding the
vector E lies along the turns thus becoming a curled field; curl E Is
a straight vector on the axis of the solenoid, equal to -mu dH/dt. E
and H work hand-in-hand.
John says: I'll put it more bluntly: E causes H and H causes E in
contradiction of the above Benjolism. E in the primary causes current
in the primary causes H in the core where mu converts it to B, whose
time rate of change infuses every coil of the secondary with the field
E whose integral with dL produces the voltage difference between the
ends of the coil.
Consequently I do not support the assertion that "E does not cause H
and H does not cause E" and yes, I am surprised that anybody would say
such a thing.
Post by Bill Miller
Or perhaps this is another example of "Two people divided by the same
language."
All the best...
Bill
John Polasek
p***@ic.ac.uk
2010-06-16 14:21:18 UTC
Permalink
Post by Bill Miller
So, here is a challenge for you. Use your FDTD software to construct a
closely spaced Parallel Plate capacitor. Apply AC to it and then use the
FDTD to "measure" the magnetic field between the plates that is supposedly
caused by the dD/dt componet in Maxwell.
A quick question: what is it you mean by D? Do you insist that

D = epsilon_0 E

only, or do you allow the very common usage

D = epsilon_0 E + P

where P is a polarization?
--
---------------------------------+---------------------------------
Dr. Paul Kinsler
Blackett Laboratory (Photonics) (ph) +44-20-759-47734 (fax) 47714
Imperial College London, ***@physics.org
SW7 2AZ, United Kingdom. http://www.qols.ph.ic.ac.uk/~kinsle/
Bill Miller
2010-06-16 21:34:54 UTC
Permalink
Post by p***@ic.ac.uk
Post by Bill Miller
So, here is a challenge for you. Use your FDTD software to construct a
closely spaced Parallel Plate capacitor. Apply AC to it and then use the
FDTD to "measure" the magnetic field between the plates that is supposedly
caused by the dD/dt componet in Maxwell.
A quick question: what is it you mean by D? Do you insist that
D = epsilon_0 E
only, or do you allow the very common usage
D = epsilon_0 E + P
where P is a polarization?
In free space and many simple environments -- like dry air --, there is no
(or minimal) discernable polarization. So, for general discussions, I prefer
D = epsilon_0*E

Depending on the dielectric material, and the frequencies and amplitudes
involved, the degree and type of dielectric polarization can act in strange
and wonderful ways to make calculations and analysis "messy."

All the best,

Bill Miller
.
Post by p***@ic.ac.uk
--
---------------------------------+---------------------------------
Dr. Paul Kinsler
Blackett Laboratory (Photonics) (ph) +44-20-759-47734 (fax) 47714
SW7 2AZ, United Kingdom. http://www.qols.ph.ic.ac.uk/~kinsle/
p***@ic.ac.uk
2010-06-17 10:34:17 UTC
Permalink
Post by Bill Miller
Post by p***@ic.ac.uk
A quick question: what is it you mean by D? Do you insist that
D = epsilon_0 E
only, or do you allow the very common usage
D = epsilon_0 E + P
where P is a polarization?
In free space and many simple environments -- like dry air --, there is no
(or minimal) discernable polarization. So, for general discussions, I prefer
D = epsilon_0*E
Depending on the dielectric material, and the frequencies and amplitudes
involved, the degree and type of dielectric polarization can act in strange
and wonderful ways to make calculations and analysis "messy."
OK. I assume this means you define D = epsilon_0 E + P?
I want to know how you define the symbol D, not what its value
may (or may not) be in this or that circumstance.

Because I can merge a current J from curl B = dt eps E + J
into my definition of D, by solving for P using dt P = J.
Then I would get a magnetic field from D (because it contains
an ordinary current), which you claim is somehow impossible.

So: in what way do you consider J to be mathematically distinguishable
from dt P? In what do you consider it to be physically distinguishable?
--
---------------------------------+---------------------------------
Dr. Paul Kinsler
Blackett Laboratory (Photonics) (ph) +44-20-759-47734 (fax) 47714
Imperial College London, ***@physics.org
SW7 2AZ, United Kingdom. http://www.qols.ph.ic.ac.uk/~kinsle/
Bill Miller
2010-06-17 14:25:58 UTC
Permalink
<***@ic.ac.uk> wrote in message news:9aape7-***@ph-kinsle.qols.ph.ic.ac.uk...
<snip>
Post by p***@ic.ac.uk
Post by Bill Miller
Depending on the dielectric material, and the frequencies and amplitudes
involved, the degree and type of dielectric polarization can act in strange
and wonderful ways to make calculations and analysis "messy."
OK. I assume this means you define D = epsilon_0 E + P?
You assume too much.

D in a general form is: epsilon * epsilon_0*E where epsilon_0 is the free
space permittivity, epsilon is the permittivity of the medium in question
and E is the E field. This is a standard definition.
Post by p***@ic.ac.uk
I want to know how you define the symbol D, not what its value
may (or may not) be in this or that circumstance.
Because I can merge a current J from curl B = dt eps E + J
into my definition of D, by solving for P using dt P = J.
Then I would get a magnetic field from D (because it contains
an ordinary current), which you claim is somehow impossible.
Free space and (to high degree) dry air is not polarizable. In over 100
years of looking between the free-space/air separated plates of a capacitor
no one has successfully measured a magnetic field that is either
"associated" with or "caused" by D.

This is not to say that if the dielectric material is polarizable, that no
magnetic field exists. In such a medium, magnetic fields can and do exist.
This is a well-known phenomenon with (reasonably) well-understood causes and
parameters.

But that's not what I'm talking about.

All the best,

Bill Miller

<snip>
Salmon Egg
2010-06-17 16:21:05 UTC
Permalink
Post by Bill Miller
<snip>
Post by p***@ic.ac.uk
Post by Bill Miller
Depending on the dielectric material, and the frequencies and amplitudes
involved, the degree and type of dielectric polarization can act in strange
and wonderful ways to make calculations and analysis "messy."
OK. I assume this means you define D = epsilon_0 E + P?
You assume too much.
D in a general form is: epsilon * epsilon_0*E where epsilon_0 is the free
space permittivity, epsilon is the permittivity of the medium in question
and E is the E field. This is a standard definition.
It is not that simple. permittivity is a TENSOR usually represent as a 3
x 3 matrix for anisotropic material (crystals). This concept is useful
when it comes to piezoelectricity and electrooptics. There also are
higher order tensors used to represent nonlinearity of materials.
Post by Bill Miller
Post by p***@ic.ac.uk
I want to know how you define the symbol D, not what its value
may (or may not) be in this or that circumstance.
Because I can merge a current J from curl B = dt eps E + J
into my definition of D, by solving for P using dt P = J.
Then I would get a magnetic field from D (because it contains
an ordinary current), which you claim is somehow impossible.
Free space and (to high degree) dry air is not polarizable. In over 100
years of looking between the free-space/air separated plates of a capacitor
no one has successfully measured a magnetic field that is either
"associated" with or "caused" by D.
Again, I have difficulty believing this statement. If it is not from
displacement, and there is no transport of charge through a dielectric,
how can you use a variable vacuum capacitor to tune an rf circuit? A
resonant circuit converts electric energy into magnetic energy and back
again.
Post by Bill Miller
This is not to say that if the dielectric material is polarizable, that no
magnetic field exists. In such a medium, magnetic fields can and do exist.
This is a well-known phenomenon with (reasonably) well-understood causes and
parameters.
But that's not what I'm talking about.
All the best,
Bill Miller
<snip>
--
An old man would be better off never having been born.
Benj
2010-06-17 18:54:57 UTC
Permalink
Post by Salmon Egg
Post by Bill Miller
Free space and (to high degree) dry air is not polarizable. In over 100
years of looking between the free-space/air separated plates of a capacitor
no one has successfully measured a magnetic field that is either
"associated" with or "caused" by D.
Again, I have difficulty believing this statement. If it is not from
displacement, and there is no transport of charge through a dielectric,
how can you use a variable vacuum capacitor to tune an rf circuit? A
resonant circuit converts electric energy into magnetic energy and back
again.
Why do you think there must be charge transport through the vacuum of
a vacuum capacitor? There certainly is force through that space. And
that force (E field) certainly acts upon the charges at both sides of
the gap. Thus there is a transport of electrical activity through the
vacuum space of the capacitor (and it tunes an RF circuit). No
problem. But what Bill is saying is that given that D field (or E
field as it's a vacuum) which is transporting force, nobody has
measured any magnetic field due to that electric field changing.
People have indeed found magnetic fields there and other people have
done the calculation showing that these fields are the result of
currents flowing in the capacitor plates and lead wires. The Maxwell
causality equations explicitly show that a magnetic field can ONLY be
created by a current or a time rate of change of current. A time rate
of change of D does NOT create a magnetic field. OF course we do know
that D and H are related and a changing current ALSO creates an
electric field. Hence it is clear that it is the CURRENT in a resonant
circuit that creates the magnetic energy and not the changing D field
of the capacitor.
Bill Miller
2010-06-17 22:48:35 UTC
Permalink
Post by Salmon Egg
Post by Bill Miller
<snip>
Post by p***@ic.ac.uk
Post by Bill Miller
Depending on the dielectric material, and the frequencies and amplitudes
involved, the degree and type of dielectric polarization can act in strange
and wonderful ways to make calculations and analysis "messy."
OK. I assume this means you define D = epsilon_0 E + P?
You assume too much.
D in a general form is: epsilon * epsilon_0*E where epsilon_0 is the free
space permittivity, epsilon is the permittivity of the medium in question
and E is the E field. This is a standard definition.
It is not that simple. permittivity is a TENSOR usually represent as a 3
x 3 matrix for anisotropic material (crystals). This concept is useful
when it comes to piezoelectricity and electrooptics. There also are
higher order tensors used to represent nonlinearity of materials.
I have absolutely no problem with this statement. But I'm not talking about
crystals, or piezoelectrics, or electrooptics or hysteresis or nonlinearity.
Materials that have one or more of these properties can and do exhibit
magnetic characteristics.

But free space permittivity has been tested from picovolts to megavolts and
from DC to daylight (and beyond). The permittivity of free space is a
constant with no nonlinearities and no hysteresis. A free space permittivity
tensor is a pretty boring sight!
Post by Salmon Egg
Post by Bill Miller
Post by p***@ic.ac.uk
I want to know how you define the symbol D, not what its value
may (or may not) be in this or that circumstance.
Because I can merge a current J from curl B = dt eps E + J
into my definition of D, by solving for P using dt P = J.
Then I would get a magnetic field from D (because it contains
an ordinary current), which you claim is somehow impossible.
Free space and (to high degree) dry air is not polarizable. In over 100
years of looking between the free-space/air separated plates of a capacitor
no one has successfully measured a magnetic field that is either
"associated" with or "caused" by D.
Again, I have difficulty believing this statement. If it is not from
displacement, and there is no transport of charge through a dielectric,
how can you use a variable vacuum capacitor to tune an rf circuit? A
resonant circuit converts electric energy into magnetic energy and back
again.
This is where it seems that your shoelaces get tied together. Yes, in a tank
circuit energy goes bouncing from E to H to E. In that regard it is a very
close analogue to a clock pendulum or a spring and weight that exchanges
kinetic energy for potential energy for kinetic...

It is ever sooooo tempting to extend this analogy to EM waves in free space.
Unfortunately, transverse waves do not perform the same way as longitudinal
waves. (Please see the recent thread on this)
Post by Salmon Egg
Post by Bill Miller
This is not to say that if the dielectric material is polarizable, that no
magnetic field exists. In such a medium, magnetic fields can and do exist.
This is a well-known phenomenon with (reasonably) well-understood causes and
parameters.
But that's not what I'm talking about.
All the best,
Bill Miller
<snip>
--
An old man would be better off never having been born.
No way! Any day that begins with waking up can't be all bad! Especially if
you consider the personal alternative.

Bill
Salmon Egg
2010-06-18 02:20:28 UTC
Permalink
Post by Bill Miller
Post by Salmon Egg
Post by Bill Miller
<snip>
Post by p***@ic.ac.uk
OK. I assume this means you define D = epsilon_0 E + P?
You assume too much.
D in a general form is: epsilon * epsilon_0*E where epsilon_0 is the free
space permittivity, epsilon is the permittivity of the medium in question
and E is the E field. This is a standard definition.
It is not that simple. permittivity is a TENSOR usually represent as a 3
x 3 matrix for anisotropic material (crystals). This concept is useful
when it comes to piezoelectricity and electrooptics. There also are
higher order tensors used to represent nonlinearity of materials.
I have absolutely no problem with this statement. But I'm not talking about
crystals, or piezoelectrics, or electrooptics or hysteresis or nonlinearity.
Materials that have one or more of these properties can and do exhibit
magnetic characteristics.
But free space permittivity has been tested from picovolts to megavolts and
from DC to daylight (and beyond). The permittivity of free space is a
constant with no nonlinearities and no hysteresis. A free space permittivity
tensor is a pretty boring sight!
I do not think that we have a fundamental disagreement. My problem with
what has been written, presuming that I read it correctly, is the
statement somewhere in this exchange, that says displacement current
does not produce a magnetic field. The whole rationale behind Maxwell's
invention of displacement current is to keep continuity of current even
in the absence of charge transfer.
<more snip>

So to the bottom line: Why does displacement current not produce
magnetic field?

Bill
--
An old man would be better off never having been born.
Bill Miller
2010-06-18 03:05:47 UTC
Permalink
"Salmon Egg" <***@sbcglobal.net> wrote in message news:SalmonEgg-***@news60.forteinc.com...
<big snip>>
Post by Salmon Egg
So to the bottom line: Why does displacement current not produce
magnetic field?
Bill
Yes... this is the bottom line and an excellent question.

The answer has to do with the basic idea of causality: If A causes B, then A
*must* occur before B.

You may want to think about that for a while. Try to come up with any
exceptions -- ever.

In Maxwell's equation delcrossH = J +dD/dt, H, J and D are all occurring
at the same time. Therefore, according to the basic tenet of causality,
*none* of those three "ingredients" can cause any or all of the other two.

Put another way, Maxwell's Equations are descriptive but not causal.

Once you accept that concept, then it is possible to start looking for what
really is the cause. Jefimenko took one approach. Panofsky and Phillips took
another. Both arrived at the same set of equations showing that E and H are
caused by the same "stuff." That stuff is charges and the motion of charges.

In an EM wave, why do time-varying E and H always occur together? Why are
they always phase locked? Because they are both caused simultaneously by the
same sources.

If this does not make you uncomfortable , then I would be surprised. For me,
it took several readings -- with lots of thought in between -- for it to
sink in. Then I said, "OH S**T!" (awakening my wife at 3 AM) as I realized
that the basic concept of electromagnetism; the concept that I first learned
att age 15 when I built my first radio, was just plain wrong.

All the best...

Bill
Post by Salmon Egg
An old man would be better off never having been born.
Salmon Egg
2010-06-18 04:09:48 UTC
Permalink
Post by Bill Miller
<big snip>>
Post by Salmon Egg
So to the bottom line: Why does displacement current not produce
magnetic field?
Bill
Yes... this is the bottom line and an excellent question.
The answer has to do with the basic idea of causality: If A causes B, then A
*must* occur before B.
You may want to think about that for a while. Try to come up with any
exceptions -- ever.
In Maxwell's equation delcrossH = J +dD/dt, H, J and D are all occurring
at the same time. Therefore, according to the basic tenet of causality,
*none* of those three "ingredients" can cause any or all of the other two.
Put another way, Maxwell's Equations are descriptive but not causal.
Once you accept that concept, then it is possible to start looking for what
really is the cause. Jefimenko took one approach. Panofsky and Phillips took
another. Both arrived at the same set of equations showing that E and H are
caused by the same "stuff." That stuff is charges and the motion of charges.
In an EM wave, why do time-varying E and H always occur together? Why are
they always phase locked? Because they are both caused simultaneously by the
same sources.
If this does not make you uncomfortable , then I would be surprised. For me,
it took several readings -- with lots of thought in between -- for it to
sink in. Then I said, "OH S**T!" (awakening my wife at 3 AM) as I realized
that the basic concept of electromagnetism; the concept that I first learned
att age 15 when I built my first radio, was just plain wrong.
All the best...
Bill
Post by Salmon Egg
An old man would be better off never having been born.
Causality is a red herring! Self-consistency is a better guide, Why is
an advanced potential as good for obtaining electromagnetic fields? Does
that mean that the past is caused by the future? Read up on stuff like
this in Feynman's Lectures on Physics. I will stick the Maxwell
Equations including delcrossH = J +dD/dt. I am content with knowing
what the magnetic field is when I know what J and D are as functions of
time and position.

E and H vary together because there also is tje equation, in your
notation delcrossE = -dB/dt.

Bill
--
An old man would be better off never having been born.
Bill Miller
2010-06-18 14:05:34 UTC
Permalink
"Salmon Egg" <***@sbcglobal.net> wrote in message news:SalmonEgg-***@news60.forteinc.com...
<snip>
Post by Salmon Egg
I am content with knowing
what the magnetic field is when I know what J and D are as functions of
time and position.
If this works so well, can you please explain why no one has ever been able
to measure the magnetic field between the plates of a capacitor that is
supposedly caused by the changing displacement current?

After that, please use Maxwell's Equations to explain why Lentz's Law works
as it does.
Post by Salmon Egg
E and H vary together because there also is tje equation, in your
notation delcrossE = -dB/dt.
Bill
--
They are time-synchronous, so one cannot cause the other

All the best,

Bill also.
Post by Salmon Egg
An old man would be better off never having been born.
Salmon Egg
2010-06-18 16:53:06 UTC
Permalink
Post by Bill Miller
They are time-synchronous, so one cannot cause the other
During the day, when I sit outside, I usually warm up in synchrony with
the sun. Thus, my warmth does not cause the sun to appear. Similarly,
the sun can not be the cause of my warmth because the appearance of the
sun and my warm feeling are synchronous.

Do I have that right?

Bill
--
An old man would be better off never having been born.
Bill Miller
2010-06-18 19:46:57 UTC
Permalink
Post by Salmon Egg
Post by Bill Miller
They are time-synchronous, so one cannot cause the other
During the day, when I sit outside, I usually warm up in synchrony with
the sun. Thus, my warmth does not cause the sun to appear. Similarly,
the sun can not be the cause of my warmth because the appearance of the
sun and my warm feeling are synchronous.
Do I have that right?
Exactly right.

Chat with you another time.
Post by Salmon Egg
Bill
--
An old man would be better off never having been born.
Salmon Egg
2010-06-18 20:36:34 UTC
Permalink
Post by Bill Miller
Post by Salmon Egg
Post by Bill Miller
They are time-synchronous, so one cannot cause the other
During the day, when I sit outside, I usually warm up in synchrony with
the sun. Thus, my warmth does not cause the sun to appear. Similarly,
the sun can not be the cause of my warmth because the appearance of the
sun and my warm feeling are synchronous.
Do I have that right?
Exactly right.
Chat with you another time.
I rest my case!

Bill
--
An old man would be better off never having been born.
John Polasek
2010-06-18 04:27:49 UTC
Permalink
Post by Bill Miller
<big snip>>
Post by Salmon Egg
So to the bottom line: Why does displacement current not produce
magnetic field?
if you look through the literature you will see that there are a
number of citations in which they calculate a magnetic field from
displacement current. It would help your case if you cited an
experiment that tested this proposition.
Post by Bill Miller
Post by Salmon Egg
Bill
Yes... this is the bottom line and an excellent question.
The answer has to do with the basic idea of causality: If A causes B, then A
*must* occur before B.
You may want to think about that for a while. Try to come up with any
exceptions -- ever.
In Maxwell's equation delcrossH = J +dD/dt, H, J and D are all occurring
at the same time. Therefore, according to the basic tenet of causality,
*none* of those three "ingredients" can cause any or all of the other two.
Put another way, Maxwell's Equations are descriptive but not causal.
Once you accept that concept, then it is possible to start looking for what
really is the cause. Jefimenko took one approach. Panofsky and Phillips took
another. Both arrived at the same set of equations showing that E and H are
caused by the same "stuff." That stuff is charges and the motion of charges.
In an EM wave, why do time-varying E and H always occur together? Why are
they always phase locked? Because they are both caused simultaneously by the
same sources.
No, it's because space has the impedance Z that E and H share:
Z = sqrt(mu/eps),
just as space has the velocity
c = sqrt(1/eps*mu).

The two are related by a sort of Ohms Law:
H amp turns/m = (E volts/m) / Z ohms.

In the case of a resistor with current flowing through it, is it the
voltage that is the cause, or is it the current? Without the resistor
there would be no action.
Post by Bill Miller
If this does not make you uncomfortable , then I would be surprised. For me,
it took several readings -- with lots of thought in between -- for it to
sink in. Then I said, "OH S**T!" (awakening my wife at 3 AM) as I realized
that the basic concept of electromagnetism; the concept that I first learned
att age 15 when I built my first radio, was just plain wrong.
All the best...
Bill
Post by Salmon Egg
An old man would be better off never having been born.
John Polasek
Bill Miller
2010-06-18 14:14:17 UTC
Permalink
Post by John Polasek
Post by Bill Miller
<big snip>>
Post by Salmon Egg
So to the bottom line: Why does displacement current not produce
magnetic field?
if you look through the literature you will see that there are a
number of citations in which they calculate a magnetic field from
displacement current. It would help your case if you cited an
experiment that tested this proposition.
Please see D.F. Bartlett, "Conduction Current and the magnetic field in a
circular capacitor." Am. J. Physics. 58, 1168-1172 (1990) In this, Bartlett
lists numerous futile attempts to measure this magnetic field "caused" by
Displacement Current.
Post by John Polasek
Post by Bill Miller
Post by Salmon Egg
Bill
Yes... this is the bottom line and an excellent question.
The answer has to do with the basic idea of causality: If A causes B, then A
*must* occur before B.
You may want to think about that for a while. Try to come up with any
exceptions -- ever.
In Maxwell's equation delcrossH = J +dD/dt, H, J and D are all occurring
at the same time. Therefore, according to the basic tenet of causality,
*none* of those three "ingredients" can cause any or all of the other two.
Put another way, Maxwell's Equations are descriptive but not causal.
Once you accept that concept, then it is possible to start looking for what
really is the cause. Jefimenko took one approach. Panofsky and Phillips took
another. Both arrived at the same set of equations showing that E and H are
caused by the same "stuff." That stuff is charges and the motion of charges.
In an EM wave, why do time-varying E and H always occur together? Why are
they always phase locked? Because they are both caused simultaneously by the
same sources.
Z = sqrt(mu/eps),
just as space has the velocity
c = sqrt(1/eps*mu).
H amp turns/m = (E volts/m) / Z ohms.
In the case of a resistor with current flowing through it, is it the
voltage that is the cause, or is it the current? Without the resistor
there would be no action.
You have stated the conditions correctly. But in order for E to cause H to
cause E...etc. It would be necessary for E and H to be out of phase. But
that would mean that the E/H ratio would have to be a complex number.

The impedance of space is 377 Ohm; not 377 +jX.

The conclusion is clear. E and H cannot be mutually causal.

All the best,

Bill
Post by John Polasek
Post by Bill Miller
If this does not make you uncomfortable , then I would be surprised. For me,
it took several readings -- with lots of thought in between -- for it to
sink in. Then I said, "OH S**T!" (awakening my wife at 3 AM) as I realized
that the basic concept of electromagnetism; the concept that I first learned
att age 15 when I built my first radio, was just plain wrong.
All the best...
Bill
Post by Salmon Egg
An old man would be better off never having been born.
John Polasek
John Polasek
2010-06-18 17:54:55 UTC
Permalink
Post by Bill Miller
Post by John Polasek
Post by Bill Miller
Post by Salmon Egg
So to the bottom line: Why does displacement current not produce
magnetic field?
if you look through the literature you will see that there are a
number of citations in which they calculate a magnetic field from
displacement current. It would help your case if you cited an
experiment that tested this proposition.
Please see D.F. Bartlett, "Conduction Current and the magnetic field in a
circular capacitor." Am. J. Physics. 58, 1168-1172 (1990) In this, Bartlett
lists numerous futile attempts to measure this magnetic field "caused" by
Displacement Current.
The only article I found they were looking for a magnetic field inside
a coax, using a frequency of 695 Hz. It sounds more like a prank. If
you have something more specific why don't you quote it.
Post by Bill Miller
Post by John Polasek
Post by Bill Miller
Post by Salmon Egg
Bill
In an EM wave, why do time-varying E and H always occur together? Why are
they always phase locked? Because they are both caused simultaneously by the
same sources.
Z = sqrt(mu/eps),
just as space has the velocity
c = sqrt(1/eps*mu).
H amp turns/m = (E volts/m) / Z ohms.
In the case of a resistor with current flowing through it, is it the
voltage that is the cause, or is it the current? Without the resistor
there would be no action.
You have stated the conditions correctly. But in order for E to cause H to
cause E...etc. It would be necessary for E and H to be out of phase. But
that would mean that the E/H ratio would have to be a complex number.
The impedance of space is 377 Ohm; not 377 +jX.
E and H are logically 180° out of phase which can be seen if E and H
are sine waves. Their operations are two derivatives apart, so that
the sine wave of one becomes -omega^2 sine of the other.
Simple analogy: voltage in a capacitor is time integral of current
1/Cint[idt, while the voltage in an inductor is the time derivative
of the current Ldi/dt, 2 derivatives ahead of the capacitor.
In AC parlance, Z = jwL and Z = 1/jwC and Z = R.
At resonance the sum of Z+Z equals 0.
Solve to find w^2 =1/LC.
Post by Bill Miller
The conclusion is clear. E and H cannot be mutually causal.
All the best,
Bill
snip
Enough already of this tutorial.
"He who learns against his will is of the same opinion still"
Post by Bill Miller
Post by John Polasek
John Polasek
John Polasek
Bill Miller
2010-06-18 21:35:48 UTC
Permalink
<snip>
Post by John Polasek
Post by Bill Miller
You have stated the conditions correctly. But in order for E to cause H to
cause E...etc. It would be necessary for E and H to be out of phase. But
that would mean that the E/H ratio would have to be a complex number.
The impedance of space is 377 Ohm; not 377 +jX.
E and H are logically 180° out of phase which can be seen if E and H
are sine waves. Their operations are two derivatives apart, so that
the sine wave of one becomes -omega^2 sine of the other.
Simple analogy: voltage in a capacitor is time integral of current
1/Cint[idt, while the voltage in an inductor is the time derivative
of the current Ldi/dt, 2 derivatives ahead of the capacitor.
In AC parlance, Z = jwL and Z = 1/jwC and Z = R.
At resonance the sum of Z+Z equals 0.
Solve to find w^2 =1/LC.
Like Salmon Egg, you are lettting an inappropriate analogy get in the way.

Let's take your postulate that the electric field and the magnetic field are
180 degrees apart. Let us define the maximum electric field value as E and
the maximum magnetic field value as H. We agree that they are
time-synchronous. And they are both sine waves.

At time T_0, let us agree that E = 0 and H = 0. Now let us move forward in
time by 30 degrees in sine wave time. The electric field amplitude will be
0.5E and the magnetic field amplitude will be - 0.5E. Move forward in time
to 90 degrees. The electric field amplitude is now E and the magnetic field
amplitude is - H.

Question 1: If the electric field and themagnetic field are exchanging
energy, how is it that the energy contained in both the electric field and
the magnetic field have both increased simultaneously? Where did the energy
come from?

We can now proceed to time t advanced by 150 degrees. Now the electric
field magnitude has decrease to 0.5E and the magnetic field has decreased to
*0.5H.

Question 2: If the electric field and themagnetic field are exchanging
energy, how is it that the energy contained in both the electric field and
the magnetic field have both decreased simultaneously? Where did the energy
go?

All the best,

Bill
glen herrmannsfeldt
2010-06-18 21:54:08 UTC
Permalink
Bill Miller <***@yahoo.com> wrote:
(snip)
Post by Bill Miller
Let's take your postulate that the electric field and the magnetic field are
180 degrees apart. Let us define the maximum electric field value as E and
the maximum magnetic field value as H. We agree that they are
time-synchronous. And they are both sine waves.
At time T_0, let us agree that E = 0 and H = 0. Now let us move forward in
time by 30 degrees in sine wave time. The electric field amplitude will be
0.5E and the magnetic field amplitude will be - 0.5E. Move forward in time
to 90 degrees. The electric field amplitude is now E and the magnetic field
amplitude is - H.
At T_0, but where?
Post by Bill Miller
Question 1: If the electric field and themagnetic field are exchanging
energy, how is it that the energy contained in both the electric field and
the magnetic field have both increased simultaneously? Where did the energy
come from?
In the case of a finite system, this question makes sense.
As an analogy, a pendulum passes energy between potential and
kinetic, and an LC filter between electrostatic (in the C)
and magnetostatic (in the L). That also applies to EM waves
in a closed resonant cavity (which is very similar to the LC case).

But consider a single frequency sine wave, that is, sin(wt)
where t is from -infinity to +infinity. More specifically,
sin(wt-kx), as a wave in space and time. Now the E energy
and B energy are not changing with time. They move in space,
but the total E energy and total B energy doesn't change with time.
(Actually, the total energies are infinite, so you need something
more like an energy density.)
Post by Bill Miller
We can now proceed to time t advanced by 150 degrees. Now the electric
field magnitude has decrease to 0.5E and the magnetic field has decreased to
*0.5H.
Question 2: If the electric field and themagnetic field are exchanging
energy, how is it that the energy contained in both the electric field and
the magnetic field have both decreased simultaneously? Where did the energy
go?
In a close box, they don't decrease simultaneously. In free space,
they move but don't decrease.

-- glen
Benj
2010-06-20 05:42:40 UTC
Permalink
Mr. Egg said: "I will continue to respond to those posters who might
gain some
knowledge from my posts. At the same time, I will try to gain some
knowledge from other posters."

Hey, that means it's time for ME to step in and enlighten everyone!
Post by glen herrmannsfeldt
Question 1: If the electric field and the magnetic field are exchanging
energy, how is it that the energy contained in both the electric field and
the magnetic field have both increased simultaneously? Where did the energy
come from?
Obviously the electric and magnetic fields are NOT exchanging energy!
Post by glen herrmannsfeldt
In the case of a finite system, this question makes sense.
As an analogy, a pendulum passes energy between potential and
kinetic, and an LC filter between electrostatic (in the C)
and magnetostatic (in the L). That also applies to EM waves
in a closed resonant cavity (which is very similar to the LC case).
Now, as it turns out, ALL of the above examples are true. Energy is
passed back and forth between two storage mechanisms that form a
resonance. And in EACH case the energy in one mode is maximum when the
other mode is zero and vice versa. Example: is a pendulum at the
bottom of the swing when potential energy is zero (we define it as
such) and velocity and hence kinetic energy is maximum. At the top of
the swing when motion reverses direction velocity is zero (and hence
kinetic energy is zero but potential energy is maximum. It it the same
for all other examples.

The common error is to assume that this same effect occurs between E
and H in EM waves in free space. It does NOT! Fact is E and H are
launched at the speed of light from the same source: namely a current
somewhere in space.
Post by glen herrmannsfeldt
But consider a single frequency sine wave, that is, sin(wt)
where t is from -infinity to +infinity. More specifically,
sin(wt-kx), as a wave in space and time. Now the E energy
and B energy are not changing with time. They move in space,
but the total E energy and total B energy doesn't change with time.
(Actually, the total energies are infinite, so you need something
more like an energy density.)
A lot of hand-waving signifying nothing.

1. Infinity does not exist in reality.
2. sin(wt-kx) from + to - infinity is non-physical nonsense.
3. Energy (Poynting vector) does indeed change with time over a
surface. Sin(wt-kx) IS a function of time. Duh.
4. Total integrated energy may or may not change with time. Usually
does change with time. Usually increases.
Think about it. A wave is expanding from it's source. The energy
integral over all space of that wave increases
as wave expands. Where is the energy coming from? The Wave SOURCE
is continually supplying it!
5. Obviously "infinite" energy is nonsense and totally non-physical.

Hence there is absolutely nothing of value in the above
"explanation"!
Post by glen herrmannsfeldt
In a close box, they don't decrease simultaneously. In free space,
they move but don't decrease.
Sure they vary. They are functions of time, remember? You've just
used a lot of words to confuse yourself. Maybe Mr. Egg can explain it
to you? But then he thinks that 180 "out of phase" means something.
But the two fields still attain zero and maximum at the same time. His
"180" degrees is simply a change in the definition of DIRECTION it
changes nothing with respect to causality. The above examples of
energy exchange have a shift of 90 degrees NOT 180! That is why when
one is maximum the other is zero...go ahead, plot it out yourself.

Here. Say you've got the sun. And on earth you have two pots of water
under two magnifying glasses. An eclipse unblocks the sun and both
pots start to boil. Mr Egg comes up with the theory that the two pots
are "talking" to each other and each is causing the other to boil. But
we note that this is impossible because both pots boil at the same
time! To "talk" would mean faster than light communication which is
impossible. Hence the ONLY theory that can stand is the one that the
energy came from the sun as the "cause" which simultaneously caused
BOTH pots to boil!

Note the major lack of thought in Mr. Egg's idea that the "warmth" and
the "appearance of the sun" are simultaneous hence the sun can't cause
warmth. Nonsense. The sun sends out rays that after a time reach earth
where they cause both warmth and the "appearance" of the perception of
an object (sun) in the sky. Mr. Egg argues that therefore the
"appearance" in the sky causes his "warmth". Total nonsense. His
"warmth" does not cause the apparition in the sky nor does that
apparition cause his warmth. BOTH are caused by the radiation from the
actual sun so many miles away. But interestingly one can deduce some
very practical rules. One can notice, for example, that whenever there
is that apparition of a solar object in the sky, you are going to feel
some warmth! So clearly the two events are CONNECTED. They do not
however (REPEAT DO NOT!) "cause" each other!

Got it?
Salmon Egg
2010-06-19 02:54:46 UTC
Permalink
Post by Bill Miller
Like Salmon Egg, you are lettting an inappropriate analogy get in the way.
Let's take your postulate that the electric field and the magnetic field are
180 degrees apart. Let us define the maximum electric field value as E and
the maximum magnetic field value as H. We agree that they are
time-synchronous. And they are both sine waves.
I certainly would not consider this a postulate. To startoff, it is
incorrect! Think standing wave. Even better, think standing wave in a
perfectly conductive wall resonator. Such an excited resonator can be
approximated rather well. What causes the electric and magnetic fields
to persist if there are on charges acting on the resonator's internal
volume.
Post by Bill Miller
At time T_0, let us agree that E = 0 and H = 0. Now let us move forward in
time by 30 degrees in sine wave time. The electric field amplitude will be
0.5E and the magnetic field amplitude will be - 0.5E. Move forward in time
to 90 degrees. The electric field amplitude is now E and the magnetic field
amplitude is - H.
Question 1: If the electric field and themagnetic field are exchanging
energy, how is it that the energy contained in both the electric field and
the magnetic field have both increased simultaneously? Where did the energy
come from?
There was another Miller, maybe Bob Miller who posted here from time to
time. He was a PE. I respected his posts. I think I must have confused
Bill Miller with him.

I am going to drop out of this thread by no longer responding to Bill
Miller until there is an indication of better understanding of EM
theory. I will continue to respond to those posters who might gain some
knowledge from my posts. At the same time, I will try to gain some
knowledge from other posters.

Bill
--
An old man would be better off never having been born.
Bill Miller
2010-06-19 19:30:02 UTC
Permalink
Post by Salmon Egg
Post by Bill Miller
Like Salmon Egg, you are lettting an inappropriate analogy get in the way.
Let's take your postulate that the electric field and the magnetic field are
180 degrees apart. Let us define the maximum electric field value as E and
the maximum magnetic field value as H. We agree that they are
time-synchronous. And they are both sine waves.
I understand and accept that Salmon Egg will no longer be replying to my
posts. But others may see his erroneous comments and so I cannot leave them
unanswered.
Post by Salmon Egg
I certainly would not consider this a postulate. To startoff, it is
incorrect!
This is not *my* postulate. John P submitted this as an idea for
cosideration. It has a certain level of plausibility since with this model,
the ratio of E to H is a real number -- 377.

Think standing wave. Even better, think standing wave in a
Post by Salmon Egg
perfectly conductive wall resonator.
Think that this is not what we are talking about. We are discussing EM waves
in Free Space. We are not talking about EM in a resonant cavity. We are not
talking about LC Tank circuits.

WE are talking about *free space.* It should come as no surprise to anyone
that fields and waves in free space are not the same as fields and waves in
a resonant device.
Post by Salmon Egg
Such an excited resonator can be
approximated rather well. What causes the electric and magnetic fields
to persist if there are on charges acting on the resonator's internal
volume.
Complelely accurate. Totally irrelevent to a discussion of free space..

All the best,

Bill
p***@ic.ac.uk
2010-06-21 12:31:29 UTC
Permalink
Post by Bill Miller
Question 1: If the electric field and themagnetic field are exchanging
energy, how is it that the energy contained in both the electric field and
the magnetic field have both increased simultaneously? Where did the energy
come from?
I'm not going to claim that the E and H fields are exchanging energy.
But the situation has both temporal _and_ spatial behavior: the time
derivative of D is linked to the curl (a spatial derivative) of H
(as with dtB and curl E).

The energy at a given point may be constant (for cw fields); but that
still allows the case where energy flows in from one side and the same
amount exits the other.
--
---------------------------------+---------------------------------
Dr. Paul Kinsler
Blackett Laboratory (Photonics) (ph) +44-20-759-47734 (fax) 47714
Imperial College London, ***@physics.org
SW7 2AZ, United Kingdom. http://www.qols.ph.ic.ac.uk/~kinsle/
Bill Miller
2010-06-22 13:33:27 UTC
Permalink
Post by p***@ic.ac.uk
Post by Bill Miller
Question 1: If the electric field and themagnetic field are exchanging
energy, how is it that the energy contained in both the electric field and
the magnetic field have both increased simultaneously? Where did the energy
come from?
I'm not going to claim that the E and H fields are exchanging energy.
We agree.
Post by p***@ic.ac.uk
But the situation has both temporal _and_ spatial behavior: the time
derivative of D is linked to the curl (a spatial derivative) of H
(as with dtB and curl E).
We agree
Post by p***@ic.ac.uk
The energy at a given point may be constant (for cw fields); but that
still allows the case where energy flows in from one side and the same
amount exits the other.
We agree, provided that we recognize that unless the point is infinitely
small (and consequentlty is not real!) there must be a time delay between
the entry and exiting of this energy..

All the best,

Bill
Post by p***@ic.ac.uk
--
---------------------------------+---------------------------------
Dr. Paul Kinsler
Blackett Laboratory (Photonics) (ph) +44-20-759-47734 (fax) 47714
SW7 2AZ, United Kingdom. http://www.qols.ph.ic.ac.uk/~kinsle/
p***@ic.ac.uk
2010-06-24 10:14:37 UTC
Permalink
Post by Bill Miller
Post by p***@ic.ac.uk
The energy at a given point may be constant (for cw fields); but that
still allows the case where energy flows in from one side and the same
amount exits the other.
We agree, provided that we recognize that unless the point is infinitely
small (and consequentlty is not real!) there must be a time delay between
the entry and exiting of this energy..
Typically you'd assume a small box: the total energy being the integral
of the energy density over the box volume, and the total flux being
the sum of fluxes through the sides. In steady state, you'd expect
no net flux and a constant total energy.

I'm not sure how you intend to label the contributions so as to track
them in one side of the box and thence out the other. If you want to
see a time delay, you need to look at transient behaviour (and thus
will no longer be in the steady state).
--
---------------------------------+---------------------------------
Dr. Paul Kinsler
Blackett Laboratory (Photonics) (ph) +44-20-759-47734 (fax) 47714
Imperial College London, ***@physics.org
SW7 2AZ, United Kingdom. http://www.qols.ph.ic.ac.uk/~kinsle/
John Polasek
2010-06-20 19:42:13 UTC
Permalink
Post by Bill Miller
Post by John Polasek
Post by Bill Miller
<big snip>>
Post by Salmon Egg
So to the bottom line: Why does displacement current not produce
magnetic field?
if you look through the literature you will see that there are a
number of citations in which they calculate a magnetic field from
displacement current. It would help your case if you cited an
experiment that tested this proposition.
Please see D.F. Bartlett, "Conduction Current and the magnetic field in a
circular capacitor." Am. J. Physics. 58, 1168-1172 (1990) In this, Bartlett
lists numerous futile attempts to measure this magnetic field "caused" by
Displacement Current.
Post by John Polasek
Post by Bill Miller
Post by Salmon Egg
Bill
Bill
Post by Salmon Egg
An old man would be better off never having been born.
John Polasek
Here is a way to see, right in Maxwell's equations, how the electric
and magnetic fields depend on each other, linked by a common
impedance Z, and not requiring a common external source such as your
charges.
1. Curl H = dD/dt = e0 Edot
2. Curl E = -dB/dt = -mu Hdot
Curl E/Curl H = -muHdot/e0Edot = Z^2 Hdot/Edot
Equating numerators and denominators we get
Curl E = Z*Hdot E depends on H
Curl H = -Edot/Z H depends on E
Maxwell's equations do not explicitly deal with impedance, but E and H
need something to get their teeth into and it is Z = E/H.
John Polasek
Bill Miller
2010-06-21 02:13:21 UTC
Permalink
Post by John Polasek
Post by Bill Miller
Post by John Polasek
Post by Bill Miller
<big snip>>
Post by Salmon Egg
So to the bottom line: Why does displacement current not produce
magnetic field?
if you look through the literature you will see that there are a
number of citations in which they calculate a magnetic field from
displacement current. It would help your case if you cited an
experiment that tested this proposition.
Please see D.F. Bartlett, "Conduction Current and the magnetic field in a
circular capacitor." Am. J. Physics. 58, 1168-1172 (1990) In this, Bartlett
lists numerous futile attempts to measure this magnetic field "caused" by
Displacement Current.
Post by John Polasek
Post by Bill Miller
Post by Salmon Egg
Bill
Bill
Post by Salmon Egg
An old man would be better off never having been born.
John Polasek
Here is a way to see, right in Maxwell's equations, how the electric
and magnetic fields depend on each other, linked by a common
impedance Z, and not requiring a common external source such as your
charges.
1. Curl H = dD/dt = e0 Edot
2. Curl E = -dB/dt = -mu Hdot
Curl E/Curl H = -muHdot/e0Edot = Z^2 Hdot/Edot
Equating numerators and denominators we get
Curl E = Z*Hdot E depends on H
Curl H = -Edot/Z H depends on E
Maxwell's equations do not explicitly deal with impedance, but E and H
need something to get their teeth into and it is Z = E/H.
John Polasek
Not quite. Instead of saying "E depends on H" we can say "E is associated
with H." This is true, but does not require nor imply causality.

In my "Mathematician" thread, please see Fred's quote and comments re
Griffiths statements on Jefimenko. Please pay particular attention to what
Fred says about the nature of these equations. Descriptive or causal?

Although he and I are having what may amount to a nomenclature disagreement
about these equations, it seems clear that we both agree that these are
*causal* equations for E and H.

If I have mis-interpreted Fred's position, I apologize and look forward to
his (always) courteous explanation.

All the best

Bill
.
John Polasek
2010-06-21 03:52:12 UTC
Permalink
Post by Bill Miller
Post by John Polasek
Here is a way to see, right in Maxwell's equations, how the electric
and magnetic fields depend on each other, linked by a common
impedance Z, and not requiring a common external source such as your
charges.
1. Curl H = dD/dt = e0 Edot
2. Curl E = -dB/dt = -mu Hdot
Curl E/Curl H = -muHdot/e0Edot = Z^2 Hdot/Edot
Equating numerators and denominators we get
Curl E = Z*Hdot E depends on H
Curl H = -Edot/Z H depends on E
Maxwell's equations do not explicitly deal with impedance, but E and H
need something to get their teeth into and it is Z = E/H.
John Polasek
Not quite. Instead of saying "E depends on H" we can say "E is associated
with H." This is true, but does not require nor imply causality.
In my "Mathematician" thread, please see Fred's quote and comments re
Griffiths statements on Jefimenko. Please pay particular attention to what
Fred says about the nature of these equations. Descriptive or causal?
Although he and I are having what may amount to a nomenclature disagreement
about these equations, it seems clear that we both agree that these are
*causal* equations for E and H.
If I have mis-interpreted Fred's position, I apologize and look forward to
his (always) courteous explanation.
All the best
Bill
We can write a matrix to relate the dots to the curls: and
Using row vectors we have the matrix Z, -1/Z in Ascii graphics below:

(Hdot Edot) (Z 0) = (Curl E Curl H)
(0 -1/Z)

The .dot and Curl vectors are related by the constant Z matrix.
A 1% change in Hdot will give a 1% change in Curl E. They are not only
associated; they are locked.

I just added what Maxwell failed to do, but it wasn't there for him,
as he worked before mu0 or eps0 were established and he only had c.
I'm sure he was working in the barren medium of cgs E&M whose
equations B = H, D = E, are only tautologies, which could could not
produce the conclusions that I have above.

In the resistor, voltage, current example, which one is the cause?
Does it make a difference? They are equally culpable, being joined by
the common impedance R.
John Polasek
Bill Miller
2010-06-22 01:35:37 UTC
Permalink
Post by Bill Miller
<snip>>
In the resistor, voltage, current example, which one is the cause?
Does it make a difference?
In this example, it makes no difference at all.

Once again... we are *supposed to be* talking about an EM wave in free
space. The characteristics of an EM wave in free space have nothing to do
with a circuit consisting of Voltage, Current and a resistor.


All the best,

Bill
Benj
2010-06-21 05:34:46 UTC
Permalink
Here is a way to see,  right in Maxwell's equations, how the electric
and magnetic fields depend on each other,  linked by a common
impedance Z, and not requiring a common external source such as your
charges.
1. Curl H = dD/dt = e0 Edot
2. Curl E = -dB/dt = -mu Hdot
Curl E/Curl H = -muHdot/e0Edot = Z^2 Hdot/Edot
Equating numerators and denominators we get
Curl E = Z*Hdot         E depends on H
Curl H = -Edot/Z   H  depends on E
Maxwell's equations do not explicitly deal with impedance, but E and H
need something to get their teeth into and it is Z = E/H.
John Polasek- Hide quoted text -
Note that time (t) in equations 1 and 2 is the SAME time. This
immediately means the equation is not causal. However it still IS an
equation. That means that the left side is EQUAL TO the right side.
Hence, as you note, the values are "locked". One can use the word
"associated" too but they are more than just related. It is an
equation so one side is equal to the other side.

But the common mistake is that everybody forgets that equal does NOT
mean "causes". In the usual use of Maxwell's equations, everyone
ignores the fact that all effects propagate at the speed of light or
thereabouts. Not just EM waves, mind you, but B fields and E fields as
well! This is called a "retarded" description when it's take into
account. Few do this because quite frankly it's a major pain in the
ass to do so. But THAT is the reality. Hence If E and H are "causing"
each other they must be delayed or "retarded" from each other. This is
not the case in 1,2 above.

The only conclusion one can draw from this is that since E and H occur
at the same time in a propagating wave, they cannot be causing each
other. The thing that PRECEDES the E and H fields is a current which
is the actual source of the fields. BOTH fields at the same time! And
therefore it is no surprise that the values of fields are "locked" to
each other by a relationship (equation). But "locked" does not imply
"causes". It's simple as that.

Note that for your resistor example, circuit theory is an
approximation to field theory and one of the approximations always
used it to ignore retardation! All values are assumed to occur at the
same time (such as the current around a loop) unless delayed by
circuit elements. Look at the equation V = I Z. both V and I have the
same identical time parameter. Hence one cannot cause the other! But
they are "locked". I don't want to say that V causes I (even though
it sort of does: consider a transmission line) because that whole
issue has already been removed by the assumptions used in circuit
theory. Hence such arguments with respect to causality are null and
void.
John Polasek
2010-06-21 18:36:22 UTC
Permalink
Post by Benj
Here is a way to see,  right in Maxwell's equations, how the electric
and magnetic fields depend on each other,  linked by a common
impedance Z, and not requiring a common external source such as your
charges.
1. Curl H = dD/dt = e0 Edot
2. Curl E = -dB/dt = -mu Hdot
Curl E/Curl H = -muHdot/e0Edot = Z^2 Hdot/Edot
Equating numerators and denominators we get
Curl E = Z*Hdot         E depends on H
Curl H = -Edot/Z   H  depends on E
Maxwell's equations do not explicitly deal with impedance, but E and H
need something to get their teeth into and it is Z = E/H.
John Polasek- Hide quoted text -
Note that time (t) in equations 1 and 2 is the SAME time. This
immediately means the equation is not causal. However it still IS an
equation. That means that the left side is EQUAL TO the right side.
Hence, as you note, the values are "locked". One can use the word
"associated" too but they are more than just related. It is an
equation so one side is equal to the other side.
But the common mistake is that everybody forgets that equal does NOT
mean "causes". In the usual use of Maxwell's equations, everyone
ignores the fact that all effects propagate at the speed of light or
thereabouts. Not just EM waves, mind you, but B fields and E fields as
well! This is called a "retarded" description when it's take into
account. Few do this because quite frankly it's a major pain in the
ass to do so. But THAT is the reality. Hence If E and H are "causing"
each other they must be delayed or "retarded" from each other. This is
not the case in 1,2 above.
The only conclusion one can draw from this is that since E and H occur
at the same time in a propagating wave, they cannot be causing each
other. The thing that PRECEDES the E and H fields is a current which
is the actual source of the fields.
In free space there is no current as the source of the fields; that
happens only in a plasma.
Post by Benj
BOTH fields at the same time! And
therefore it is no surprise that the values of fields are "locked" to
each other by a relationship (equation). But "locked" does not imply
"causes". It's simple as that.
You conveniently left out the matrix I created that shows how these
fields are locked together so I'll reproduce it here just for
reference
(Hdot Edot) (Z 0) = (Curl E Curl H)
(0 -1/Z)
The .dot and Curl vectors are related by the constant Z matrix.
A 1% change in Hdot will give a 1% change in Curl E. They are not only
associated; they are locked.
Neither author (M or J ) is paying attention to Z (one couldn't and
one didn't).
Do you think the expression
Z = sqrt(mu/eps) = 377 ohms
is some sort of joke? In fact it is practically a direct obligation of
a modern author to discuss and dispose of Z. It is the direct key as
to how E & H are conjoined. It is as valid and as meaningful as
c = sqrt(1/mu*eps)
Impedance Z is what causes E&H to work together and to be mutual
causes and effects for each other. As Wiki says, Z sets amplitudes of
E&H with respect to each other. H = E/Z.
Of course you can write equations that recognizetime delays for
certain purposes (making sure you're prepared to say why you need
them). But that doesn't allow you to say that time precedence is a
requirement to establish cause and effect.
Or, more importantly, saying that lack of time precedence proclaims
that there is no connection.

John Polasek
Benj
2010-06-22 04:35:57 UTC
Permalink
Post by John Polasek
BOTH fields at the same time!  And
therefore it is no surprise that the values of fields are "locked" to
each other by a relationship (equation). But "locked" does not imply
"causes". It's simple as that.
You conveniently left out the matrix I created that shows how these
fields are locked together so I'll reproduce it here just for
reference
(Hdot  Edot) (Z      0)  = (Curl E  Curl H)
                     (0 -1/Z)
The .dot and Curl vectors are related by the constant Z matrix.
A 1% change in Hdot will give a 1% change in Curl E. They are not only
associated; they are locked.
Cripes! I SAID they are "locked"! What more do you want?
Post by John Polasek
Neither author (M or J ) is paying attention to Z (one couldn't and
one didn't).  
Do you think the expression
        Z = sqrt(mu/eps) = 377 ohms
is some sort of joke? In fact it is practically a direct obligation of
a modern author to discuss and dispose of Z. It is the direct key as
to how E & H are conjoined. It is as valid and as  meaningful as
        c = sqrt(1/mu*eps)
Impedance Z is what causes E&H to work together and to be mutual
causes and effects for each other. As Wiki says, Z sets amplitudes of
E&H with respect to each other. H = E/Z.
Well, Wiki in spite of my best efforts probably STILL says that E
causes H and vice versa! We know that E and H are equal to each other
by some factor. And Z sets that ratio. But that still does not say
that that two equal things CAUSE each other. Now indeed a change in Z
will CAUSE a change in that ratio. Imagine a wave bopping along with a
certain ratio between E and H. It runs into a dielectric or other
material. Suddently the ratio changes. One could say that the Z has
changed the ratio. And indeed it has. And if you'd like to confuse
yourself, examine the situation very carefully and you'll find that
the WAY the ratio is changed is that the fields incident upon the
material create additional fields that are added to the incident
fields to produce a new total field that represents the new ratio. So
in this case one indeed CAN say the E and H are "creating each other"
to a degree! But what about the simultaneous thing? Doesn't that
preclude such causality? No it doesn't because these fields are NOT
simultaneous! The material responds with a finite delay and hence
there is an actual time difference in the creation of new fields.

At this point you could start to get excited and say you could define
aether as a material with impedance Z (of free space) that responds
infinitely fast (unlike real materials) so therefore by such a
definition E and H could "create" each other! And, yeah, it ALMOST
works! The problem is that free space is NOT "superluminal". Inspite
of it's nearly ideal properites we know that there is no known means
to transmit action (energy, information etc.) faster than the speed of
light. Hence that trick fails.
Post by John Polasek
Of course you can write equations that recognizetime delays for
certain purposes (making sure you're prepared to say why you need
them). But that doesn't allow you to say that time precedence is a
requirement to establish cause and effect.
Or, more importantly, saying that lack of time precedence proclaims
that there is no connection.
Correct. The rule is simultaneous actions at separate points in space
cannot "cause each other". Actually they can't cause each other at a
single point in space either.
John Polasek
2010-06-22 15:18:10 UTC
Permalink
Post by Benj
Post by John Polasek
BOTH fields at the same time!  And
therefore it is no surprise that the values of fields are "locked" to
each other by a relationship (equation). But "locked" does not imply
"causes". It's simple as that.
You conveniently left out the matrix I created that shows how these
fields are locked together so I'll reproduce it here just for
reference
(Hdot  Edot) (Z      0)  = (Curl E  Curl H)
                     (0 -1/Z)
The .dot and Curl vectors are related by the constant Z matrix.
A 1% change in Hdot will give a 1% change in Curl E. They are not only
associated; they are locked.
Cripes! I SAID they are "locked"! What more do you want?
Post by John Polasek
Neither author (M or J ) is paying attention to Z (one couldn't and
one didn't).  
Do you think the expression
        Z = sqrt(mu/eps) = 377 ohms
is some sort of joke? In fact it is practically a direct obligation of
a modern author to discuss and dispose of Z. It is the direct key as
to how E & H are conjoined. It is as valid and as  meaningful as
        c = sqrt(1/mu*eps)
Impedance Z is what causes E&H to work together and to be mutual
causes and effects for each other. As Wiki says, Z sets amplitudes of
E&H with respect to each other. H = E/Z.
Well, Wiki in spite of my best efforts probably STILL says that E
causes H and vice versa! We know that E and H are equal to each other
by some factor. And Z sets that ratio. But that still does not say
that that two equal things CAUSE each other. Now indeed a change in Z
will CAUSE a change in that ratio. Imagine a wave bopping along with a
certain ratio between E and H. It runs into a dielectric or other
material. Suddently the ratio changes. One could say that the Z has
changed the ratio. And indeed it has. And if you'd like to confuse
yourself, examine the situation very carefully and you'll find that
the WAY the ratio is changed is that the fields incident upon the
material create additional fields that are added to the incident
fields to produce a new total field that represents the new ratio. So
in this case one indeed CAN say the E and H are "creating each other"
to a degree! But what about the simultaneous thing? Doesn't that
preclude such causality? No it doesn't because these fields are NOT
simultaneous! The material responds with a finite delay and hence
there is an actual time difference in the creation of new fields.
At this point you could start to get excited and say you could define
aether as a material with impedance Z (of free space) that responds
infinitely fast (unlike real materials) so therefore by such a
definition E and H could "create" each other! And, yeah, it ALMOST
works! The problem is that free space is NOT "superluminal". Inspite
of it's nearly ideal properites we know that there is no known means
to transmit action (energy, information etc.) faster than the speed of
light. Hence that trick fails.
Post by John Polasek
Of course you can write equations that recognizetime delays for
certain purposes (making sure you're prepared to say why you need
them). But that doesn't allow you to say that time precedence is a
requirement to establish cause and effect.
Or, more importantly, saying that lack of time precedence proclaims
that there is no connection.
Correct. The rule is simultaneous actions at separate points in space
cannot "cause each other". Actually they can't cause each other at a
single point in space either.
Maybe this will convince you how intimately the fields are connected
and how you have failed to discern this connection, as a consequence
of not acknowledging the action of Z = sqrt(mu/eps):
H = E/Z amp turns per meter = volts/meter/ohms
H^2 = E^2/Z^2
muH^2 = epsE^2 (1/2)
As a result of this truism, that H = E/Z, it forces the well accepted
but totally unexpected result that the magnetic energy equals the
electric energy and establishes Z as a cornerstone. Z forces them to
be equal-equal? how could they be equal if as you say they're totally
separate? If you want to ignore Z than please dispose of it, because
it is an otherwise valuable quotient of two constants that cannot be
ignored and has the units of ohms.
But this does not guarantee they are simultaneous; they most likely
take turns.
What has Jefimenko got to say about Z?
I am sorry to see so many people caught up in CGS (who like to remain
anonymous, "but it is by their works that ye shall know them") where
mu and eps are missing and there can't be a Z or anything else of
consequence for that matter. Its science by recitation.
John Polasek
Benj
2010-06-23 04:50:37 UTC
Permalink
Post by John Polasek
Maybe this will convince you how intimately the fields are connected
and how you have failed to discern this connection, as a consequence
        H = E/Z   amp turns per meter = volts/meter/ohms
        H^2 = E^2/Z^2
        muH^2 = epsE^2   (1/2)
As a result of this truism, that H = E/Z, it forces the well accepted
but totally unexpected result that the magnetic energy equals the
electric energy and establishes Z as a cornerstone. Z  forces them to
be equal-equal? how could they be equal if as you say they're totally
separate? If you want to ignore Z than please dispose of it, because
it is an otherwise valuable quotient of two constants that cannot be
ignored and has the units of ohms.
So your argument is that if I have two bank accounts and they have
equal amounts of money in each and if my rule is that each time I add
a "dollar" to one I add it to the other so they are "locked" that this
means they cannot be separate but must be the SAME thing? Nonsense.
Even if they have different units of money in them and I adjust my
units so they appear to have equal amounts of "dollars" in each one
that still doesn't make them the same thing. Obviously Z is somehow
related to the way in which E and H are generated at their SOURCE! In
other words it's an expression of the ratio of propagated E and H
generated by charges and currents.

Which is a very interesting idea now that I think of it! It implies
that Z is not some property of "free space" that creates a ratio
between E and H as an "impedance of free space", but rather is a
property of fields and charges creating the propagating wave. Hmmm.

Now if you want to argue that E and H are actually the SAME thing (as
some do) then obviously causality takes a new turn. Because now E and
H happen at the same time but they are the SAME THING! Thus, causality
isn't an issue. So if you want to argue that there is only ONE field
which transforms back and forth as far as meaurments are concerned
(has to do with the frames in which E and H are measured) bu Lorentz
transformations, well, that rather changes the picture from what
Jefimenko is arguing!
glen herrmannsfeldt
2010-06-23 07:22:07 UTC
Permalink
Benj <***@iwaynet.net> wrote:
(snip)
Post by Benj
that still doesn't make them the same thing. Obviously Z is somehow
related to the way in which E and H are generated at their SOURCE! In
other words it's an expression of the ratio of propagated E and H
generated by charges and currents.
I believe that Z can be related to E and B (or H in free space)
without any reference to the charges and currents creating the wave.

(snip)
Post by Benj
Now if you want to argue that E and H are actually the SAME thing (as
some do) then obviously causality takes a new turn. Because now E and
H happen at the same time but they are the SAME THING! Thus, causality
isn't an issue. So if you want to argue that there is only ONE field
which transforms back and forth as far as meaurments are concerned
(has to do with the frames in which E and H are measured) bu Lorentz
transformations, well, that rather changes the picture from what
Jefimenko is arguing!
OK, say your EM wave was created by an oscillating magnetic monopole.
Would you be able to tell the difference just looking at the wave,
even billions of years after it was created? I would guess not.

-- glen
Benj
2010-06-23 16:46:37 UTC
Permalink
Post by glen herrmannsfeldt
OK, say your EM wave was created by an oscillating magnetic monopole.
Would you be able to tell the difference just looking at the wave,
even billions of years after it was created?  I would guess not.
It would be nice if you could since nobody has observed a magnetic
monopole (yet). Of course you can tell certain things about the source
of a wave. Given that there is a transform relationship between the
radiation pattern and the current source creating a given wave,
presumeably one can calculate backwards from an antenna pattern
measured in all of space to the current distribution creating that
pattern. But I'm sure there is more to it than that. For one thing,
the energy into space is not just the far field waves. There are also
near field energies as well. Of course those die off quickly with
distance meaning that after billions of years that part is long gone.
John Polasek
2010-06-24 03:25:17 UTC
Permalink
Post by Benj
Post by John Polasek
Maybe this will convince you how intimately the fields are connected
and how you have failed to discern this connection, as a consequence
        H = E/Z   amp turns per meter = volts/meter/ohms
        H^2 = E^2/Z^2
        muH^2 = epsE^2   (1/2)
As a result of this truism, that H = E/Z, it forces the well accepted
but totally unexpected result that the magnetic energy equals the
electric energy and establishes Z as a cornerstone. Z  forces them to
be equal-equal? how could they be equal if as you say they're totally
separate? If you want to ignore Z than please dispose of it, because
it is an otherwise valuable quotient of two constants that cannot be
ignored and has the units of ohms.
So your argument is that if I have two bank accounts and they have
equal amounts of money in each and if my rule is that each time I add
a "dollar" to one I add it to the other so they are "locked" that this
means they cannot be separate but must be the SAME thing? .
I am appalled at the superficial level of your reasoning, and that's
after I take umbrage at the way you patronize me with your puerile
analogies.
Post by Benj
Even if they have different units of money in them and I adjust my
units so they appear to have equal amounts of "dollars" in each oneNonsense
that still doesn't make them the same thing. Obviously Z is somehow
related to the way in which E and H are generated at their SOURCE!
I asked you above whether the 377 Ohms of space was a joke ort you
have no idea how radio waves are generated. The power is carried in a
tuned LC circuit and fed to an antenna that meshes with the "ether"
and the job of the designer is to make the output impedance match that
of space, Z = 377 ohms (or fraction thereof depending on the antenna).
If it is matched, all of the power goes out, but if it is not matched
a fraction of it comes back with each cycle leading to excess currents
and heat, called standing waves. When the cycles leave the antenna
into free space they are on the something and it appears that you do
think so. It's quite clear they are on their own own, lashed together
by the impedance of space sqrt(mu/eps).
Have you ever taken anything apart to see what makes it work? Maybe
not. But hethat's what you should do with mu and eps of space and ask
yourself "What are these OHMS for?"
Countless years of effort have been expended deriving dimensional
ratios such as Planck's length not even having demonstrable merit.
Given two fundamental constants, mu amd eps. it is inescapable that
their products and ratio, c and Z, respectively would be recognized as
fundamental new quantities. Investigators missing the impedance of
space are severely handicapped.
Since you prefer trivial examples, you can think of someone blowing a
large bubble in any which way he pleases, but after the breeze carries
it away you can see it wobbling and undulating and obeying its own
differential equations governed by the coefficients of the bubble's
body.
Post by Benj
other words it's an expression of the ratio of propagated E and H
generated by charges and currents.
Which is a very interesting idea now that I think of it! It implies
that Z is not some property of "free space" that creates a ratio
between E and H as an "impedance of free space", but rather is a
property of fields and charges creating the propagating wave. Hmmm.
Now if you want to argue that E and H are actually the SAME thing
No, one is volts per meter and the other is amp turns per meter. Their
quotient is volts/amperes, somewhat obscured as the cognomen Ohms.
(I can't cover the whole topic here in the depth which it deserves).
Such units are not needed in CGS of which you appear to be a devotee
because you suggested that E&H might be the same thing. By the same
token stress and strain are not the same thing.
Post by Benj
as some do) then obviously causality takes a new turn. Because now E and
H happen at the same time but they are the SAME THING! Thus, causality
isn't an issue. So if you want to argue that there is only ONE field
which transforms back and forth as far as meaurments are concerned
(has to do with the frames in which E and H are measured) bu Lorentz
transformations, well, that rather changes the picture from what
Jefimenko is arguing!
John Polasek
p***@ic.ac.uk
2010-06-21 13:56:21 UTC
Permalink
Post by Bill Miller
The answer has to do with the basic idea of causality: If A causes B, then A
*must* occur before B.
You may want to think about that for a while. Try to come up with any
exceptions -- ever.
In Maxwell's equation delcrossH = J +dD/dt, H, J and D are all occurring
at the same time. Therefore, according to the basic tenet of causality,
*none* of those three "ingredients" can cause any or all of the other two.
Note the "d/dt" acting on D; we have

d/dt D = curl H - J

or in the frequency domain

-i omega D(omega) = curl H(omega) - J(omega)

==> D(omega) = -(curl H(omega) - J(omega))/i omega

Thus we can see that D will causally dependent on H, J in the
Kramers-Kronig sense -- the LHS is analytic in the upper half
plane and so Titchmarsh's theorum holds (as do the Kramers-Kronig
relations).

I think that in a sense it is "barely causal", in that the H and J
spectra will have to be well behaved and not diverge at high frequencies.
The 1/omega dependence means that the response function linking
(curl H(omega) - J(omega)) to D(omega) is the bare step function,
nad not something smoother, as is usual.

You are welcome, of course, to have alternative definitions of causality:
in particular, you seem to want a response function to be zero for
t<=0 (rather than the KK/Titchmarsh t<0). I'm not a priori averse to
this, but it'd be nice of you could back it up with some rigorous
mathematics to make it as useful a definition as that which gives the
KK relations. Bt would it really give different results except in
some exotic mathematical edge cases?
--
---------------------------------+---------------------------------
Dr. Paul Kinsler
Blackett Laboratory (Photonics) (ph) +44-20-759-47734 (fax) 47714
Imperial College London, ***@physics.org
SW7 2AZ, United Kingdom. http://www.qols.ph.ic.ac.uk/~kinsle/
Bill Miller
2010-06-22 14:46:19 UTC
Permalink
Post by p***@ic.ac.uk
Post by Bill Miller
The answer has to do with the basic idea of causality: If A causes B, then A
*must* occur before B.
You may want to think about that for a while. Try to come up with any
exceptions -- ever.
In Maxwell's equation delcrossH = J +dD/dt, H, J and D are all occurring
at the same time. Therefore, according to the basic tenet of causality,
*none* of those three "ingredients" can cause any or all of the other two.
Note the "d/dt" acting on D; we have
d/dt D = curl H - J
or in the frequency domain
-i omega D(omega) = curl H(omega) - J(omega)
==> D(omega) = -(curl H(omega) - J(omega))/i omega
Thus we can see that D will causally dependent on H, J in the
Kramers-Kronig sense -- the LHS is analytic in the upper half
plane and so Titchmarsh's theorum holds (as do the Kramers-Kronig
relations).
I think that in a sense it is "barely causal"
Isn't this like being "a little bit pregnant?" Either something causes
something else or it doesn't. (Unless QM enters the picture. Then all "bets
seem to be off. See below.)
Post by p***@ic.ac.uk
in that the H and J
spectra will have to be well behaved and not diverge at high frequencies.
The 1/omega dependence means that the response function linking
(curl H(omega) - J(omega)) to D(omega) is the bare step function,
nad not something smoother, as is usual.
in particular, you seem to want a response function to be zero for
t<=0 (rather than the KK/Titchmarsh t<0). I'm not a priori averse to
this, but it'd be nice of you could back it up with some rigorous
mathematics to make it as useful a definition as that which gives the
KK relations.
Actually, the QM folks seem to toss all of the above out the window with
"entanglement," in which B occurs *before* A. IOW the response function
will *never* be zero! That kind of thinking defies any sort of "rigorous"
mathematics. :-)
Post by p***@ic.ac.uk
Bt would it really give different results except in
some exotic mathematical edge cases?
Well, in the case of Maxwell and EM, Jefimenko's work provides a clear
explanation for Lentz's law and tells us why a changing H appears to *cause*
E (Faraday's Law) but no one has ever been able to measure the results of
the inverse proposition that E *causes* H.

If you have not done so, I recommend that you take a hard look at his
derivation. I am unaware of any successful challenges to it. Alternately,
take a look at Panofsky & Phillips. They derived these causal equations
using an entirely different method.

All the best,

Bill Miller
Post by p***@ic.ac.uk
--
---------------------------------+---------------------------------
Dr. Paul Kinsler
Blackett Laboratory (Photonics) (ph) +44-20-759-47734 (fax) 47714
SW7 2AZ, United Kingdom. http://www.qols.ph.ic.ac.uk/~kinsle/
p***@ic.ac.uk
2010-06-22 15:26:40 UTC
Permalink
Post by Bill Miller
Post by p***@ic.ac.uk
I think that in a sense it is "barely causal"
Isn't this like being "a little bit pregnant?" Either something causes
something else or it doesn't.
No, nothing like "a little bit pregnant". I'm saying it is (mathematically)
causal, but it is on the edge of allowed causal behaviour.
Post by Bill Miller
[...]
Actually, the QM folks seem to toss all of the above out the window with
"entanglement," in which B occurs *before* A. IOW the response function
will *never* be zero! That kind of thinking defies any sort of "rigorous"
mathematics. :-)
There is quite a lot of rigorous QM. And QM is causal, in the usual
prescription: you maintain strict causality at the cost of making
probability more complicated (i.e. complex probability amplitudes,
not old-fashioned real-valued probabilities).

You can interpret QM with old-fashioned real-valued probabilities if you
like -- but then have to have non-local ("non-causal") behaviour. Take
your pick.
--
---------------------------------+---------------------------------
Dr. Paul Kinsler
Blackett Laboratory (Photonics) (ph) +44-20-759-47734 (fax) 47714
Imperial College London, ***@physics.org
SW7 2AZ, United Kingdom. http://www.qols.ph.ic.ac.uk/~kinsle/
Szczepan Bia³ek
2010-06-18 08:14:39 UTC
Permalink
Post by Salmon Egg
The whole rationale behind Maxwell's
invention of displacement current is to keep continuity of current even
in the absence of charge transfer.
There are the charge transfer but on the very short distances.
Displacement current exists only in the oscillating form.
S*
Benj
2010-06-07 06:16:27 UTC
Permalink
Post by akh2103
Hi--
I'm a recent humanities grad trying to get a grasp of EM. I can
understand/visualize sound waves propagating through a medium. But I
can't understand or visualize EM waves in the same way. I get that EM
waves are 'fundamental' and can't be broken down/explained in terms of
smaller parts (the way a sound wave can be analyzed in terms of
molecules). but i still can't 'see' EM waves. Can anybody help?
Thanks!
AH
A humanities grad trying to understand EM waves? You are doomed! Not
even Nobel prize winners (and I mean REAL ones not political ones like
Algore) have a clue how EM waves work! The posts in this thread (by
some of the most knowledgeable folks posting here) give you the hint
that nobody has a clue.

First off, it is believed today that EM waves don't travel in a medium
at all. As a humanities grad you are obviously going to have trouble
with nonsense like this. Maxwell did. He noted that there are only TWO
ways to transmit energy from one place to another: One is kinetic like
say a bullet. And the other is by waves (say ocean or sound waves). A
bullet can do that in empty space no medium required. Waves require a
medium to transmit energy. In Maxwell's time that medium was called
"Aether" and was assumed to fill all void. Modern science rejects the
idea and claims that EM waves travel through no medium at all.
Obviously nonsense, but nobody in physics is prepared to point out the
Emperor's nakedness!

Oh, but as Blackhead pointed out, it gets still worse. We know that
light for sure is not waves at all! And probably ALL EM radiation is
not waves. Light is clearly particles that some how statistically in
large numbers exhibit wave properties. People have looked into this
and it's understood to a degree. But nevertheless nobody has a CLUE as
to what exactly these light particles (called "photons") are or why
they produce statistics with wave properties. To see how this works
see Wikipedia on the double slit experiment.

So the bottom line is you've asked a question well beyond the
capabilities of our primitive science and while as you can see there
are MANY people willing to try to impress you with the facts they DO
know, none of them really knows the answer to your question. Because
as you know for sound, for waves to propagate there must be a medium
that can be stressed and that stress must be able to be carried from
one element of the medium to the next. Since no medium is admitted for
EM radiation there simply cannot be any wave propagation. And indeed
the discovery of photons pretty much verifies that. However, at the
same time statistically solutions to wave equations and wave-like
phenomena are observed even on SINGLE photons. How this works, however
is still a mystery.

Maybe you should just write the great American novel instead...
glen herrmannsfeldt
2010-06-07 07:14:53 UTC
Permalink
Benj <***@iwaynet.net> wrote:
(snip)
Post by Benj
A humanities grad trying to understand EM waves? You are doomed! Not
even Nobel prize winners (and I mean REAL ones not political ones like
Algore) have a clue how EM waves work! The posts in this thread (by
some of the most knowledgeable folks posting here) give you the hint
that nobody has a clue.
I have to find the right quote in the Feynman lectures.
There are some at the beginning of vol. 2, but I think I remember
even more than that.

(snip)
Post by Benj
Oh, but as Blackhead pointed out, it gets still worse. We know that
light for sure is not waves at all! And probably ALL EM radiation is
not waves. Light is clearly particles that some how statistically in
large numbers exhibit wave properties. People have looked into this
and it's understood to a degree. But nevertheless nobody has a CLUE as
to what exactly these light particles (called "photons") are or why
they produce statistics with wave properties. To see how this works
see Wikipedia on the double slit experiment.
Oh, I always thought it was waves that showed particle-like
statistics at low intesities. So, it is the other way around!

But as for sound waves through a gas, it doesn't seem so obvious
to me that you can easily connect the collisions of gas particles
into wave motion. And, for that matter, Newton got it wrong when
he tried to do just that.

For a solid or liquid it isn't so hard to use the continuous
medium approximation for wavelengths much longer than the atomic
spacing. That isn't quite as easy, at least for me, in the
case of a gas.
Post by Benj
So the bottom line is you've asked a question well beyond the
capabilities of our primitive science and while as you can see there
are MANY people willing to try to impress you with the facts they DO
know, none of them really knows the answer to your question. Because
as you know for sound, for waves to propagate there must be a medium
that can be stressed and that stress must be able to be carried from
one element of the medium to the next. Since no medium is admitted for
EM radiation there simply cannot be any wave propagation. And indeed
the discovery of photons pretty much verifies that. However, at the
same time statistically solutions to wave equations and wave-like
phenomena are observed even on SINGLE photons. How this works, however
is still a mystery.
OK, a quote from section 1-5 in Feynman Lectures on Physics, vol. 2:

"You may be saying: "All this business of fluxes and curculations
is pretty abstract. There are electric fields at every point
in space; then there are these 'laws.' But what is actually
happening? Why can't you explain it, for instance, by whatever
it is that goes between the charges." Well, it depends on your
prejudices. Many physicists used to say that direct action with
nothing in between was inconceivable. (How could they find an
idea inconceivable when it had already been conceived?) They
would say: "Look, the only forces we know are the direct action
of one piece of matter on another. It is impossible that there
can be a force with nothing to transmit it." But what really
happens when we study that "direct action" of one piece of
matter right against the other? We discover that it is not one
piece right against the other; they are slightly separated, and
there are electrical forces acting on a tiny scale. Thus we find
that we are going to explain these so-called direct contact
action in terms of the picture of electical forces. It is
certainly not sensible to try to insist that an electrical force
has to look like the old, familiar, muscular push or pull when
it will turn out that that muscular pushes and pulls are going
to be interpreted as electrical forces! The only sensible
question is what is the most convenient way to look at electrical
effects. Some people prefer to represent them at interaction at
a distance of charges, and to use complicated laws. Others love
these field lines. They draw lines all the time, and feel that
writing E's and B's is too abstract. The field lines, however,
are only a crude way of describing a field, and it is very
difficult to give the correct, quantitative laws directly in
terms of field lines."

Well, there is more but maybe that gets the idea across. Oh,
and Feynman got the Nobel prize for his explanation of quantum
electrodynamics, which is what this is all about.

-- glen
Szczepan Bia³ek
2010-06-07 08:54:40 UTC
Permalink
Post by glen herrmannsfeldt
But as for sound waves through a gas, it doesn't seem so obvious
to me that you can easily connect the collisions of gas particles
into wave motion. And, for that matter, Newton got it wrong when
he tried to do just that.
For Bill and me it is obvious. Sound source kicks the molecules
periodically. "The molecules then bumps into another molecule and another."
Post by glen herrmannsfeldt
For a solid or liquid it isn't so hard to use the continuous
medium approximation for wavelengths much longer than the atomic
spacing. That isn't quite as easy, at least for me, in the
case of a gas.
In the gas the atomic spacing is much bigger. The wavelengths must be
longer. It is all.
Post by glen herrmannsfeldt
"You may be saying: "All this business of fluxes and curculations
is pretty abstract. There are electric fields at every point
in space; then there are these 'laws.' But what is actually
happening? Why can't you explain it, for instance, by whatever
it is that goes between the charges." Well, it depends on your
prejudices. Many physicists used to say that direct action with
nothing in between was inconceivable. (How could they find an
idea inconceivable when it had already been conceived?) They
would say: "Look, the only forces we know are the direct action
of one piece of matter on another. It is impossible that there
can be a force with nothing to transmit it." But what really
happens when we study that "direct action" of one piece of
matter right against the other? We discover that it is not one
piece right against the other; they are slightly separated, and
there are electrical forces acting on a tiny scale.
At contacts are the flexsible deflections.
Post by glen herrmannsfeldt
Thus we find
that we are going to explain these so-called direct contact
action in terms of the picture of electical forces. It is
certainly not sensible to try to insist that an electrical force
has to look like the old, familiar, muscular push or pull when
it will turn out that that muscular pushes and pulls are going
to be interpreted as electrical forces! The only sensible
question is what is the most convenient way to look at electrical
effects. Some people prefer to represent them at interaction at
a distance of charges, and to use complicated laws. Others love
these field lines. They draw lines all the time, and feel that
writing E's and B's is too abstract. The field lines, however,
are only a crude way of describing a field, and it is very
difficult to give the correct, quantitative laws directly in
terms of field lines."
Well, there is more but maybe that gets the idea across. Oh,
and Feynman got the Nobel prize for his explanation of quantum
electrodynamics, which is what this is all about.
What he choose, the "action at a distance" or collisions?
S*
Post by glen herrmannsfeldt
-- glen
glen herrmannsfeldt
2010-06-10 05:31:03 UTC
Permalink
"Szczepan Bia³ek" <***@wp.pl> wrote:
(snip, I wrote)
Post by Szczepan Bia³ek
Post by glen herrmannsfeldt
For a solid or liquid it isn't so hard to use the continuous
medium approximation for wavelengths much longer than the atomic
spacing. That isn't quite as easy, at least for me, in the
case of a gas.
In the gas the atomic spacing is much bigger. The wavelengths must be
longer. It is all.
If it was that easy, Newton would have gotten it right.

First, the expansion and contraction is adiabatic, and Newton
thought it was isothermal. Is that obvious? Second, oxygen
and nitrogen are diatomic, which affects the adiabatic expansion
and contraction. That, at least, wasn't obvious to me.

-- glen
Benj
2010-06-08 00:54:16 UTC
Permalink
  "You may be saying: "All this business of fluxes and circulations
   is pretty abstract.  There are electric fields at every point
   in space; then there are these 'laws.'  But what is actually
   happening?  Why can't you explain it, for instance, by whatever
   it is that goes between the charges."  Well, it depends on your
   prejudices.  Many physicists used to say that direct action with
   nothing in between was inconceivable.  (How could they find an
   idea inconceivable when it had already been conceived?)  They
   would say: "Look, the only forces we know are the direct action
   of one piece of matter on another.  It is impossible that there
   can be a force with nothing to transmit it."  But what really
   happens when we study that "direct action" of one piece of
   matter right against the other?  We discover that it is not one
   piece right against the other;  they are slightly separated, and
   there are electrical forces acting on a tiny scale.  Thus we find
   that we are going to explain these so-called direct contact
   action in terms of the picture of electrical forces.  It is
   certainly not sensible to try to insist that an electrical force
   has to look like the old, familiar, muscular push or pull when
   it will turn out that that muscular pushes and pulls are going
   to be interpreted as electrical forces!  The only sensible
   question is what is the most convenient way to look at electrical
   effects.  Some people prefer to represent them at interaction at
   a distance of charges, and to use complicated laws.  Others love
   these field lines.  They draw lines all the time, and feel that
   writing E's and B's is too abstract.  The field lines, however,
   are only a crude way of describing a field, and it is very
   difficult to give the correct, quantitative laws directly in
   terms of field lines."
Well, there is more but maybe that gets the idea across.  Oh,
and Feynman got the Nobel prize for his explanation of quantum
electrodynamics, which is what this is all about.  
I get the idea, Glen, but do you? The late Feynman was not only one of
the coolest cats on the planet (See his books of stories such as
"Surely you are joking, Mr. Feynman") but also one of the smartest.
So, OK, what did the smartest guy on the planet (after Bill Gates)
have to say about this question? In essence he's saying that it's OK
to assume that electrical and magnetic forces happen through "magic".
Nothing needs to touch. Nothing needs to intervene. Nothing produces
the forces. He's saying that the long discredited "action at a
distance" theory is still a valid thing to use (so long as you are
careful not to use the old terminology and embarrass physics.) In
other words, one of the smartest and most knowledgeable scientist to
walk the planet doesn't have any more clue about these things than we
do! I rest my case.


Of course he did get the Nobel Prize (a real one) for QED and that
theory does indeed provide a description of certain actions with
regard to light. However, it in truth does not provide any real
explanation. As Feynman might have put it succinctly: It's MAGIC! So
get over it!
Szczepan Bialek
2010-06-07 08:30:42 UTC
Permalink
Post by Benj
First off, it is believed today that EM waves don't travel in a medium
at all. As a humanities grad you are obviously going to have trouble
with nonsense like this. Maxwell did. He noted that there are only TWO
ways to transmit energy from one place to another: One is kinetic like
say a bullet. And the other is by waves (say ocean or sound waves). A
bullet can do that in empty space no medium required. Waves require a
medium to transmit energy. In Maxwell's time that medium was called
"Aether" and was assumed to fill all void. Modern science rejects the
idea and claims that EM waves travel through no medium at all.
Obviously nonsense, but nobody in physics is prepared to point out the
Emperor's nakedness!
Some know that Maxwell and Heaviside use the "Kinematic description" for the
aether.

"" It is natural to begin this discussion by considering the various
possible types of motion in themselves, leaving out of account for a time
the causes to which the initiation of motion may be ascribed; this
preliminary enquiry constitutes the science of Kinematics. - ET
Whittaker[4]


In kinematics equations no mass. But they describe something what exsists.
S*
Loading...